You are on page 1of 40

[PEDIATRICS 3A] Nephrology A

Lecturer: Joselito C. Matheus, MD


Transcriber: Patrick Angelo R. Bautista February 2020


References and Legends Table 509-1 Other Causes of Red Urine
• {⏯} Recordings + {💻} Powerpoint + {📕} Lecture Guide and Manual HEME POSITIVE
• {📖} Nelson Textbook of Pediatrics 20th Ed Hemoglobin
Myoglobin
• {📌} Transcriber’s Note
HEME NEGATIVE
Dyes (Vegetable / Fruit)
Table of Contents Drugs
Beets
Chloroquine
I. Hematuria 1 Blackberries
Deferoxamine
II. Post-Infectious (Streptococcal) Glomerulonephritis 2 Food and candy coloring
Ibuprofen
III. Proteinuria and Nephrotic Syndrome 3 Rhubarb
Iron sorbitol
IV. Urinary Tract Infection 6 Metronidazole
Metabolites
V. IGA Nephropathy 9 Nitrofurantoin
Homogentisic acid
VI. Henoch-Schonlein Purpura (HSP) Nephritis 9 Phenazopyridine (Pyridium)
Melanin
VII. Renal Venous Thrombosis 10 Pholphthalein
Methemoglobin
11 Phenothiazines
VIII. Wilms Tumor (Nephroblastoma) Porphyrin
Rifampin
IX. Reading Assignment: SLE, HUS, Interstitial Nephritis 12 Tyrosinosis
Salicylates
X. Compiled Samplex 14 Urates
Sulfasalazine
Table 509-2 Causes of Hematuria in Children
I. HEMATURIA {📖 509} UPPER URINARY TRACT DISEASE
• Gross Hematuria Isolated renal disease
Immunoglobulin (Ig) A nephropathy (Berger disease)
9 Bright red color; or brown, tea, coffee or cola-colored urine or
Alport syndrome (hereditary nephritis)
presence of clots in the urine Thin glomerular basement membrane nephropathy
§ Bright red urine – post glomerular Postinfectious GN (Poststreptococcal GN) *
§ Tea-colored urine – glomerular Membranous nephropathy
• Microscopic / Significant Hematuria Membranoproliferative GN*
9 Diagnosed through microscopy; >5 RBCs/hpf* on >2 occasions Rapidly progressive GN
9 {⏯} Range: >3 to >5 RBC/hpf Focal segmental glomerulosclerosis
Anti–glomerular basement membrane disease
§ Lower cut-off implication (>3): you will be screening for
Multisystem disease
more people and getting work-up for patient that is normal Systemic lupus erythematosus nephritis*
§ Higher cut-off implication (>5): less people screened, Henoch-Schönlein purpura nephritis
possibly miss patients who may have significant hematuria. Granulomatosis with polyangiitis (formerly Wegener granulomatosis)
• Persistent Hematuria Polyarteritis nodosa
9 Significant hematuria by dipstick test or microscopy in 3 Goodpasture syndrome
separate occasions over a span of 2-3 weeks Hemolytic-uremic syndrome
Sickle cell glomerulopathy
9 {⏯} 3 urinalyses are usually not required. 1 is enough if:
HIV nephropathy
§ (+) microscopic evidence of hematuria Tubulointerstitial disease
§ Patient is already passing out tea-colored urine Pyelonephritis Interstitial nephritis
• Tea-colored or cola-colored urine may be due to: Papillary necrosis
9 Hemoglobinuria – due to diseases causing hemolysis and the Acute tubular necrosis
subsequent release of hemoglobin Vascular
9 Myoglobinuria – due to muscle injuries like crushing injuries, Arterial or venous thrombosis
Malformations (aneurysms, hemangiomas)
sudden and excessive use of specific muscle groups causing
Nutcracker syndrome
muscle injury and some are due to medications. Hemoglobinopathy (sickle cell trait/disease)
• Dipstick / Test strips Crystalluria
9 Can detect 5-10 intact RBCs/uL or 2-5 RBC/hpf Anatomic
9 If it falls under the 5 RBC cut-off, it may or may not be Hydronephrosis
significant, so you have to redo the test. Cystic-syndromic kidney disease
9 Note: test strips (qualitative) should be correlated with Polycystic kidney disease
Multicystic dysplasia
microscopic findings (quantitative)
Tumor (Wilms, rhabdomyosarcoma, angiomyolipoma, medullary Ca)
Trauma
LOWER URINARY TRACT DISEASE
Inflammation (infectious and noninfectious) Cystitis
Urethritis
Urolithiasis
Trauma
Coagulopathy
Heavy exercise
Bladder tumor
Factitious syndrome, factitious syndrome by proxy†
*Denotes glomerulonephritides presenting with hypocomplementemia.
†Formerly Munchausen syndrome and Munchausen syndrome by proxy.

Nephrology A | 1 of 14

II. POST-INFECTIOUS GN {📖 511}


• Patients are at risk for developing:
9 Encephalopathy and/or heart failure secondary to
• Umbrella term; may be bacterial, viral, protozoan
hypertension or hypervolemia.
9 Most common cause is GABHS
• Hypertensive encephalopathy
9 Other organisms: staphylococcus, salmonella, measles,
9 Considered in patients with: blurred vision, severe headaches,
mumps, chickenpox, malaria
• PIGN is characterized by a previous infection prior to onset of
altered mental status, new seizures.
nephritic syndrome • Symptoms of Pulmonary Edema and Heart Failure
• Immune complex reaction leading to nephritis 9 Respiratory distress, orthopnea, cough
• Peripheral edema from salt and water retention
Infection ➡ Latent Period ➡ Nephritic Syndrome • Nephrotic syndrome
Latent period: • Nonspecific symptoms:
• Analogous to an incubation period 9 Malaise, lethargy, abdominal pain, flank pain
• Variable; <1 week to 6 weeks • Atypical presentations:
9 Average: 10-14 days for post-strep GN 9 Subclinical disease, severe symptoms but an absence of initial
urinary abnormalities, purpuric rash
Acute Post-Streptococcal Glomerulonephritis • The acute phase generally resolves within 6-8 wk.
• Although urinary protein excretion and hypertension usually
• Classic example of the acute nephritic syndrome characterized
normalize by 4-6 wk after onset, persistent microscopic hematuria
by the sudden onset of:
can persist for 1-2 years after the initial presentation.
9 Gross hematuria, edema, hypertension, renal insufficiency

PSGN: Etiology PSGN: Diagnosis {💻+📖+📕}


• APSGN follows infection of the throat or skin by certain • Urinalysis
9 Gross or microscopic hematuria
“nephritogenic” strains of GABHS. 2
• PSGN commonly follows: 9 Proteinuria (cut-off for nephritic: <40 mg/m /hr)
2
9 Strep pharyngitis during cold-weather months
§ >40 mg/m /hr is nephrotic
§ Serotypes: 1, 3, 4, 12, 25, 49 • Complete blood count
9 Mild normochromic anemia may be present from
9 Skin infections/ pyoderma during warm-weather months
§ Serotypes: 2, 49, 55, 57, 60 hemodilution and low-grade hemolysis.
+
• Serum electrolytes (Na, K, Ca, P, H )
+
Note: If the question is what is the etiology of PSGN, the answer 9 Hypo-Na, hyper-K, hypo-Ca, hyper-P, increased H
is immune complex reaction. • Acid-base balance (case to case basis): metabolic acidosis
• Renal function test
Group A Beta hemolytic strep is responsible for the infection prior 9 BUN/Creatinine, eGFR
but the development of PSGN is due to immune complex. § Compute using Schwartz Formula: eGFR = kL/PCr
§ k = constant; L = height; PCr = plasma creatinine
PSGN: Pathogenesis • ASO, neuraminidase, DNAse test
Theories on Renal Injury: Immune mediated 9 Antistreptolysin O (ASO) titer – commonly ↑ after a
• Trapping of circulating immune complexes in the glomeruli pharyngeal infection but rarely ↑ after strep skin infections.
• Molecular mimicry between streptococcal and renal antigens 9 Anti-deoxyribonuclease (DNAse) B level – best single
• In-situ immune complex formation between anti-streptococcal antibody titer to document cutaneous streptococcal infection
antibodies and glomerular planted antigen • Throat swab/ culture
• Direct complement activation by streptococcal antigens deposited • Skin lesion discharge culture
in the glomeruli • Serum C3, C4
9 Serum C3 level – significantly reduced in >90% of patients in
PSGN: Clinical Manifestations of Acute PSGN {💻+📖} the acute phase, and returns to normal 6-8 wk after onset
Symptom % 9 Serum CH50 – commonly depressed
Hematuria 100 9 Serum C4 – most often normal, or only mildly depressed
Proteinuria 80 • Magnetic Resonance Imaging of the brain
Edema 90 9 Indicated in patients with severe neurologic symptoms
Hypertension 60-80 9 Can demonstrate posterior reversible encephalopathy
Oliguria 10-50 syndrome in the parietooccipital areas on T2-weighted images
Dyspnea, Heart Failure <5 • Chest X-ray
Nephrotic proteinuria 4 9 Indicated in those with signs of heart failure or respiratory
Azotemia 25-40
distress, or physical exam findings of a heart gallop, decreased
Early mortality <1
breath sounds, rales, or hypoxemia.
• Most common in children ages 5-12 years
Essential for Diagnosis
9 Uncommon before the age of 3 years.
• Identification of the primary infection (History)
• The typical patient develops an acute nephritic syndrome: • Establish presence of latent period (History)
9 1-2 weeks after a streptococcal pharyngitis or • Appearance of signs and symptoms of nephritis (History + PE)
9 3-6 weeks after a streptococcal pyoderma • Evidence of previous or ongoing strep infection (Laboratory)
• The history of a specific infection may be absent, because
Majority of cases – good prognosis and asymptomatic in nature
symptoms may have been mild or have resolved without patients
receiving specific treatment or seeking the care of a medical
PSGN: Complications
provider.
• Volume overload
• The severity of kidney involvement varies from asymptomatic
• Renal failure
microscopic hematuria with normal renal function to gross
• Electrolyte imbalance
hematuria with acute renal failure.
• Hypertensive Crisis / Hypertensive Encephalopathy
• Depending on the severity of renal involvement, patients can
develop various degrees of edema, hypertension, and oliguria.

Nephrology A | 2 of 14

PSGN: Treatment {💻+📕} • Dialysis – seldom used


• Primarily supportive 9 Peritoneal Dialysis or Hemodialysis
9 Indication: progressive increase in BUN and creatinine
• Penicillin for 10 days – drug of choice*
9 To completely eradicate remaining organism
especially with uremic signs and symptoms, intractable
9 Alternatives: hypertension or intractable hyperkalemia
ST ND
§ Amoxicillin, 1 / 2 gen Cephalosporins, Macrolides
• Maintain Homeostasis – fluid and electrolytes PSGN: Prognosis {💻+📕}
9 Regulation of Fluid – prevent fluid overload • Good, >90% recover without long term sequelae
§ Diuretics: Furosemide 1-2 mg/kg/dose • Natural History of PSGN
§ Fluid Restriction: 9 Oliguric phase – presence of nephritic s/sx (2 wks)
(BSA x Insensible H2O loss + Urine Output) / 2 9 Diuretic phase – ↑ urine output, nephritic s/sx subside (2 wks)
• Anti-hypertensives 9 Resolution phase – improvement in urinalysis findings
9 Hypertension >95% for age and height (proteinuria, hematuria decreasing in intensity until normal), C3
9 Use appropriate size of cuff normalizes in 4-6 wks
• Diuretics, CCB, ACEi / ARBs, Vasodilators

III. PROTEINURIA and NEPHROTIC SYNDROME {📖 523-527}

Proteinuria {📖} 9 Podocyte Functions:


§ Structural support of the capillary loop
• Classified as either benign or pathologic; Classified as
§ Major component of glomerular filtration barrier to proteins
glomerular or tubular proteinuria
§ Synthesis and repair of glomerular basement membrane
• Normal protein excretion is defined as ≤4 mg/m2/hr
• Foot processes
• Abnormal proteinuria is defined as excretion of 4-40 mg/m2/hr
9 Extensions of the podocyte that terminate on the glomerular
• Nephrotic-range proteinuria is defined as >40 mg/m2/hr
basement membrane.
9 Interdigitate with those from adjacent podocytes and are
Nephrotic Syndrome {💻+📖}
connected by a slit called the slit diaphragm.
• Clinical manifestation of glomerular diseases associated with • Slit diaphragm
heavy (nephrotic-range) proteinuria 9 One of the major impediments to protein permeability across
2
9 *Proteinuria >3.5 g/24hr or >40 mg/m /hr the glomerular capillary wall.
9 *Urine protein:creatinine ratio >2 9 Not simple passive filters—they consist of numerous proteins
• Triad of clinical findings associated with nephrotic syndrome: that contribute to complex signaling pathways and play an
1. Hypoalbuminemia (≤2.5 g/dL) important role in podocyte function.
2. Edema 9 Important component proteins of the slit diaphragm:
3. Hyperlipidemia (cholesterol >200 mg/dL) § Nephrin, podocin, CD2AP, α-actinin 4
• Responsive to corticosteroid therapy • Podocyte injury or genetic mutations of genes producing
podocyte proteins may cause nephrotic-range proteinuria.
NS: Pathogenesis: Role of a Podocyte • In idiopathic, hereditary, and secondary forms of nephrotic
• The underlying abnormality in nephrotic syndrome is an syndrome, there are immune and nonimmune insults to the
increased permeability of the glomerular capillary wall, which podocyte that lead to:
leads to massive proteinuria and hypoalbuminemia. 9 Foot process effacement of the podocyte

• Podocyte 9 Decrease in number of functional podocytes

9 Plays a crucial role in the development of proteinuria and 9 Altered slit diaphragm integrity

progression of glomerulosclerosis. • The end result is increased protein “leakiness” across the
9 A highly differentiated epithelial cell located on the outside of glomerular capillary wall into the urinary space.
the glomerular capillary loop.

Table 526-1 Causes of Proteinuria


TRANSIENT PROTEINURIA GLOMERULAR DISEASES WITH PROTEINURIA AS A PROMINENT FEATURE
Fever Acute postinfectious glomerulonephritis (streptococcal, endocarditis,
Exercise hepatitis B or C virus, HIV)
Dehydration Immunoglobulin A nephropathy
Cold exposure Henoch-Schönlein purpura nephritis
Congestive heart failure Lupus nephritis
Seizure Serum sickness
Stress Alport syndrome
ORTHOSTATIC (POSTURAL) PROTEINURIA Vasculitic disorders
GLOMERULAR DISEASES CHARACTERIZED BY ISOLATED PROTEINURIA Reflux nephropathy
Idiopathic (minimal change) nephrotic syndrome TUBULAR DISEASES
Focal segmental glomerulosclerosis Cystinosis
Mesangial proliferative glomerulonephritis Wilson disease
Membranous nephropathy Lowe syndrome
Membranoproliferative glomerulonephritis Dent disease (X-linked recessive nephrolithiasis)
Amyloidosis Galactosemia
Diabetic nephropathy Tubulointerstitial nephritis
Sickle cell nephropathy Acute tubular necrosis
Renal dysplasia
Polycystic kidney disease
Reflux nephropathy
Drugs (penicillamine, lithium, NSAID)
Heavy metals (lead, gold, mercury)

Nephrology A | 3 of 14

Table 523-1 Methods Available to Test for Proteinuria


METHOD INDICATIONS NORMAL RANGE COMMENTS
Dipstick testing Routine screening for proteinuria Negative or trace in a concentrated False-positive test can occur if urine
performed in the office urine specimen (spgr: ≥1.020) is very alkaline (pH > 8.0) or very
concentrated (spgr: >1.025)
24 hr urine for protein and Quantitation of proteinuria (as well <100 mg/m2/24 hr or <150 mg/24 hr More accurate than spot urine
creatinine* excretion as creatinine clearances) in a documented 24 hr collection analysis; inconvenient for patient;
limited use in pediatric practice
Spot urine for protein/creatinine Semiquantitative assessment of <0.2 mg protein/mg creatinine in Simplest method to quantitate
ratio—preferably on first morning proteinuria children >2 yr old proteinuria; less accurate than
urine specimen <0.5 mg protein/mg creatinine in measuring 24 hr proteinuria
those 6–24 mo old
Microalbuminuria Assess risk of progressive <30 mg urine albumin per gram of Therapy should be intensified in
glomerulopathy in patients with creatinine on first morning urine diabetics with microalbuminuria
diabetes mellitus
*Note that in a 24 hr urine specimen, the creatinine content should be measured to determine whether the specimen is truly a 24 hr collection. The amount of
creatinine in a 24 hr specimen can be estimated as follows: females, 15-20 mg/kg; males, 20-25 mg/kg.

Table 527-2 Summary of Primary Renal Diseases That Manifest as Idiopathic Nephrotic Syndrome
FEATURES MINIMAL CHANGE FOCAL SEGMENTAL MEMBRANOUS MEMBRANOPROLIFERATIVE GLOMERULONEPHRITIS
NEPHROTIC SYNDROME GLOMERULOSCLEROSIS NEPHROPATHY Type I Type II
DEMOGRAPHICS
Age (years) 2-6, some adults 2-10, some adults 40-50 5-15 5-15
Sex 2:1 male 1.3:1 male 2:1 male Male-female Male-female
CLINICAL MANIFESTATIONS
Nephrotic syndrome 100% 90% 80% 60% * 60% *
Asymptomatic 0 10% 20% 40% 40%
proteinuria
Hematuria (microscopic 10-20% 60-80% 60% 80% 80%
or gross)
Hypertension 10% 20% early Infrequent 35% 35%
Rate of progression to Does not progress 10 yr 50% in 10-20 yr 10-20 yr 5-15 yr
renal failure
Associated conditions Usually none HIV, heroin use, sickle Renal vein thrombosis; None Partial lipodystrophy
cell disease, reflux medications; SLE;
nephropathy hepatitides B, C;
lymphoma; tumors
GENETICS
None except in Podocin, α-actinin 4, None None None
congenital TRPC6 channel, INF-
nephrotic syndrome 2, MYH-9
LABORATORY FINDINGS
Manifestations of Manifestations of Manifestations of Low complement Normal complement
nephrotic syndrome nephrotic syndrome nephrotic syndrome levels—C1, C4, C3- levels—C1, C4, low
↑ BUN in 15-30% ↑ BUN in 20-40% Normal complement C9 C3-C9
Normal complement Normal complement levels
levels levels
RENAL PATHOLOGY
Light microscopy Normal Focal sclerotic lesions Thickened GBM, spikes Thickened GBM, Lobulation
proliferation
Immunofluorescence Negative IgM, C3 in lesions Fine granular IgG, C3 Granular IgG, C3 C3 only
Electron microscopy Foot process fusion Foot process fusion Subepithelial deposits Mesangial and Dense deposits
subendothelial
deposits
REMISSION ACHIEVED AFTER 8 WEEKS OF ORAL CORTICOSTEROID THERAPY
90% 15-20% Resistant Not established/ Not established/
resistant resistant
*Approximate frequency as a cause of idiopathic nephrotic syndrome. Approximately 10% of cases of adult nephrotic syndrome are a result of various diseases
that usually manifest as acute glomerulonephritis.

Histologic Dx (Primary Disease) in Renal Biopsies of 521 children NS: Clinical Features
Minimal Change Nephrotic Syndrome 77.1% Clinical Hallmark of Nephrotic Syndrome is Edema Formation
Focal Segmental Glomerulonephritis 7.9%
Membranoproliferative Glomerulonephritis 6.2%
Others (membranous GN, mesangial GN) 8.8%
• Note: those in red show progressive disease despite treatment
and eventually leads to End-Stage Renal Disease.

Responsiveness to Steroid Treatment


• Minimal Change Nephrotic Syndrome – most sensitive
9 Sensitivity of 93.1%
9 Specificity of 72.2%
Classic Symptoms:
Clinical Classification of Childhood NS • Periorbital edema, taut skin, scrotal edema, pitting pedal edema
• Secondary (10%)
9 Related to systemic disease, drug, toxin, allergen, APSGN
Edema
• Most common presenting sx of children with Nephrotic syndrome
• Congenital Nephrotic Syndrome 2
• Massive proteinuria (>40 mg/m /hr)
• Asymmetric and dependent (influenced by gravity)

Nephrology A | 4 of 14

Proteinuria (Albuminuria) Note: For steroid sensitive nephrotic syndrome, majority will undergo
• Dipstick: 4+ albumin diuresis within 5-10 days after starting steroid treatment. Although
• Protein/Crea ratio: random urine sample – screening test only diuretics may be used to manage edema, caution has to be observed
9 >200 mg/mmol or >2 mg/dL in its use because the patient may already be in a state of volume
• 24-hour urine albumin determination: Gold standard contraction that will be aggravated by diuretic use.
2
9 Through body surface area: >40 mg/m /hr
9 Through weight: >50 mg/kg/day Other Medications
• Antacids: usually not necessary
NS: Investigation of Initial Presentation • Calcium Carbonate: for prolonged steroid use (>3 months)
• Hyperlipidemia medications: not usually required for steroid
• CBC with platelet count
sensitive NS
9 ↑ WBC – infection
• Isoniazid: for 6 months for Mantoux positive only
9 ↑ hematocrit, thrombocytosis - intravascular vol. contraction
• TB disease: treated with standard therapy
• Serum creatinine, cholesterol, TPAG
• Urinalysis
Natural Course of Steroid Sensitive NS (Minimal Change Disease)
• 24-hour creatinine clearance / eGFR
• Renal Ultrasound: Not mandatory; only if entertaining differentials • 80% will have one or more relapses
• Others: • 50% will either have frequent relapses or be steroid dependent
9 C3 (depends on the clinical features) – prognostic indicator • Increasing age = Decreasing incidence of relapses
§ Minimal change disease = normal C3 9 Frequency of relapses decrease with time with:
§ Primary type: Membranous proliferative disease = ↓ C3 § 50- 70% being relapse-free after 5 years
9 Hepatitis B Screening § 85% relapse-free at 10 years
9 Tuberculin test • Early relapse after initial treatment and short duration of remission
increase risks for subsequent relapse.
Secondary Nephrotic Syndrome • Patients with frequent relapses during childhood are more likely to
• Systemic disease with renal manifestations have disease persisting into adulthood.
• Supporting laboratory exams exist for the primary disease
9 Ex. SLE with (+) tests for ANA, anti-DNAse, anti-Sm NS: Complications
• Thrombosis
NS: Current Treatment 9 Correction of hypovolemia and hemoconcentration
• International Study of Kidney Disease in Children (ISKDC) 9 Anticoagulation for thromboembolic episodes and those at risk
• Arbeitsgemeinschaft fur Padiatrische Nephrologie (APN) • Hypovolemia (Shock)
2
• DOC: Prednisone 60 mg/m → 40 mg/m
2 9 Abdominal pain, oliguria, cold peripheries, poor pulse volume,
hypotension, hemoconcentration, usually minimal edema
Steroid Treatment Regimen: ISKDC vs. APN 9 20-25% salt poor albumin 0.5-1.0g/kg/dose over 1-2 hr infusion
• ISKDC: 8 weeks (4 weeks daily, 4 weeks alternate days) 9 Furosemide 1-2 mg/kg/dose
9 2 months intensive treatment • Hyperlipidemia
ST
§ 1 half (4 weeks daily): 60 mg/m
2 9 Dietary advice
ND
§ 2 half (4 weeks alternate): 40 mg/m
2 9 HMG-CoA reductase inhibitors are effective but experience is
• APN: 12 weeks* (6 weeks daily, 6 weeks alternate days) still limited (not usually necessary if responsive to steroids)
9 Superior than ISKDC regimen 9 ↑ risk for CVD for Non-Minimal Change Disease (NMCD) with
9 3 months intensive treatment persistently heavy proteinuria and ↑ VLDL and LDL
ST 2
§ 1 half (6 weeks daily): 60 mg/m * • Infection
ND
§ 2 half (6 weeks alternate): 40 mg/m
2 9 Relapses with complaint of abdominal pain
9 Primary peritonitis
Steroid Response § Cover for both gram (+) and (-) until cultures are available
Remission Proteinuria and edema subsides with 3 § Prophylactic oral penicillin 125-250 mg BID in an
consecutive (-) proteinuria (dipstick) edematous child
Frequent Relapse 2-3 relapses in 6 months § Streptococcus pneumoniae is most common agent
3-4 relapses in a year causing peritonitis and septicemia
Infrequent Relapse 1-2 relapses in a year
Steroid Dependent (+) remission but with relapses after tapering NS: Immunization
steroids • Live vaccines (measles, polio boosters) not given to patients on
Steroid Resistant Still unresponsive after giving intensive course corticosteroids; can be given 4 weeks after cessation of treatment
• Killed vaccine may be given anytime
Steroid-Sensitive Nephrotic Syndrome Diet • 23-polyvalent pneumococcal polysaccharide vaccine
• Balanced diet 9 Given especially if prone to S. pneumoniae infections (relapses
• Protein at 1.5-2.5 g/kg for patients with persistent proteinuria associated with URTI)
9 For growth and nitrogen balance 9 >2 years old
9 High protein diets >2.5 g/kg worsens renal symptoms 9 Variable response
• Salt restriction: 1-2 g/day (no added salt) 9 Loss of antibody titer over time
• Ensure physical activity and prevent excessive weight gain • PVC 13
• Education regarding effects of high dose steroids: • Influenza A
9 Voracious appetite, central obesity, fluid retention, 9 Adequate Ab titer maintained at 6 months post-vaccination
hypertension, diabetes mellitus, cataract • Susceptible Nephrotics
9 Varicella exposure
Edema Treatment § Zoster immunoglobulin within 72 hours from exposure
• Diuresis from treatment within 5-10 days § Acyclovir, if varicella develops
• Oral furosemide 1-3 mg/kg daily 9 Measles exposure
9 For persistent edema and weight gain of 7-10% § Gamma globulin
• Spironolactone 2-4 mg/kg daily
9 For patients with prolonged and high dose furosemide NS: Prognosis
requirements.
• Overall good prognosis for Minimal Change
• Albumin transfusion (1-2 g/kg) – should be done first before
• Guarded prognosis for other primary types (MPGN, FSGS)
diuresis to increase oncotic pressure.

Nephrology A | 5 of 14

IV. URINARY TRACT INFECTION {📖 538}


Other Risk Factors
• Congenital Functional Problems
UTI: Prevalence and Etiology 9 Important concern for these may cause recurrent UTI
• Prevalence of UTIs varies with age • Immune System of the Patient
9 During 1st yr of life – M:F ratio is 2.8-5.4: 1 9 Could be due to increase virulence and decrease in the

9 Beyond 1-2 yrs – female preponderance; M:F ratio 1:10.


defense mechanism of the patient
• Circumcision
9 In boys: most UTIs occur during the 1st yr of life; much more
9 Risk of complications outweighed the benefits in boys without
common in uncircumcised boys
UTI, but in boys with previous UTI or high grade VUR (III- V),
9 In girls: the first UTI usually occurs by the age of 5 yrs, with
the benefits outweighed the risks.
peaks during infancy and toilet training.
9 Circumcision of the male infant with VUR may be considered
• Etiology
based on an ↑ risk of UTI in boys who are not circumcised.
9 Females: E. coli (75-90%), Klebsiella, Proteus
• Bladder Bowel Dysfunction (BBD)
9 Males: E. coli, Proteus, Gram (+) 9 The risk of febrile UTI in children with VUR on continuous
9 Both Sexes: S. saprophyticus, Enterococcus antibiotic prophylaxis is greater in patients with BBD compared
9 Adenovirus, other virus – cystitis with gross hematuria with those without BBD
• Epidemiology: {💻} 9 BBD Risk Factors
9 By the age of 7 years, 8.4% of girls and 1.7% of boys has had § Day time urinary incontinence
one or more symptomatic UITs § Squatting and other holding maneuvers (Sphincter control,
9 UTI is most common in the first year of life with boys having children seating on heel, etc.)
more UTI that girls in the first 6 months of life. § Prolonged or fractionated voiding
§ ↑ Residual volumes (pre- vs. post-voiding bladder US)
UTI: Risk Factors § Constipation – directly related to UTI
§ Fecal soiling
Table 538-1 Risk Factors for Urinary Tract Infection
9 BBD Intervention
Female gender Tight clothing (underwear)
Uncircumcised male Pinworm infestation § Increasing fluid intake
Vesicoureteral reflux * Constipation § Regular or timed voiding (e.g. every 3 hours even if they
Toilet training Bacteria with P fimbriae don’t feel urgency to void)
Voiding dysfunction Anatomic abnormality (labial § Relaxation measures to avoid holding maneuvers
Obstructive uropathy adhesion) § Prevention and treatment of constipation
Urethral instrumentation Neuropathic bladder § Use of bladder and bowel diaries to assess progress
Wiping from back to front in girls Sexual activity
Bubble bath? Pregnancy
*Risk increased for clinical pyelonephritis, not cystitis
UTI: Common Uropathogens / Etiologic Agents
• Predominant organisms are gram (-) organisms:
Independent Risk Factors for Recurrence are: 9 E. coli – 70% of UTI
• Age below 6 months at first UTI 9 Others: Proteus mirabilis, Klebsiella, Enterobacter,
• Age 3-5 years Pseudomonas aeruginosa, Enterococcus
• Vesicoureteral reflex grades III, IV, V • Proteus vulgaris: for uncircumcised boys
• S. saprophyticus: acute UTI in adolescent girls

UTI: Pathogenesis
• Ascending infection
• Periurethral area contains bowel bacteria
• This is caused predominantly of E. coli in girls, and after the first 6
months of life, Proteus in boys
• In older children, gram negative bacteria colonize the periurethral
area and precedes the development of UTI
• The change from normal flora may be induced by a course of a
broad-spectrum antibiotics causing an infection
Grading of vesicoureteral reflux:
Grade I: VUR into a non-dilated ureter. Bacterial Virulence Factors Host Response to UTI
Grade II: VUR into the upper collecting system without dilation. • E. coli strains express surface • Innate host immune system
Grade III: VUR into dilated ureter and/or blunting of calyceal fornices. fimbriae (P fimbriae) • Cytokine production
Grade IV: VUR into a grossly dilated ureter. • Flagella mediated motility • Neutrophil recruitment to kill
Grade V: massive VUR, with significant ureteral dilation and tortuosity • Lipopolysaccharide production bacteria
and loss of the papillary impression. • Capsular polysaccharide • IL-6
• Hemolysins • IL-8 causes an increase in
Host Factors • E. coli compete with host cells neutrophil migration and
for nutrients including Iron activation resulting in pyuria
• Constipation – directly related to UTI
Aerobactin which is a high
• Infrequent urination affinity iron binding protein
• Flushing mechanism of urine helps in getting rid of bacteria
• Associated with holding or postponing urination UTI: Clinical Manifestations and Classification
• Normal urination in neonates is 8-12x; voids every feeding
• Immune system
3 Basic Forms of UTI
a. Pyelonephritis – involvement of the renal parenchyma
• Congenital/functional problems: may cause recurrent UTI
9 Characterized by any or all of the following:
§ Abdominal, back, or flank pain
Bacterial Virulence Factors
§ Fever – may be the only manifestation at times
• Ability to adapt
§ Malaise
• Fimbriae
§ Nausea, vomiting
§ Diarrhea (occasionally)
9 Newborns – non-specific signs and symptoms
§ Poor feeding, irritability, jaundice, weight loss
9 Pyelitis – term when no renal parenchyma involved
9 Pyelonephritic scarring – renal injury due to pyelonephritis

Nephrology A | 6 of 14

b. Cystitis – involvement of the bladder Criteria for the Diagnosis of UTI by Urine Culture
9 Dysuria, urgency, frequency, suprapubic pain, incontinence, Method of Collection Colony Count Probability of Infection
9 Malodorous urine – not specific for a UTI Suprapubic aspirate Any number >99%
9 Does not cause fever and does not result in renal injury Catheter ≥10,000 95%
Clean void ≥100,000
c. Asymptomatic Bacteriuria Boy 1 specimen Infection unlikely
9 Positive urine culture without any manifestations of infection
9 Most common in girls Girl 1 specimen 80%
9 Benign condition 2 specimens 90%
9 Does not cause renal injury except in pregnants
What may cause a false-negative culture:
• Focal Pyelonephritis (Nephronia) and Renal abscess
• Had already taken antibiotic medication
9 Less common forms
• Double voided urine sample
• Diluted urine
Clinical Presentation
• The most useful indicators of UTI in infants aged <24 months are:
UTI: Imaging Studies
9 Fever above 40 degrees
9 Previous history of UTI • To detect: Obstructive lesions, VUR, Kidney damage
9 Fever for more than 24 hours • Indications:
9 Suprapubic tenderness 9 Any complicated UTI (>3 months, congenital abnormalities)
9 Ill appearance 9 Acute pyelonephritis
9 No other source of fever 9 Bacteriuria before 1 year of age
9 Uncircumcised in boys 9 Hypertension
9 Combined predictors of fever exceeding 39°C for >48 hours 9 Abdominal mass
without another source of fever were more useful than 9 Decreased renal concentrating ability
individual findings. 9 Recurrent cystitis in boys
9 The presence of abdominal pain, back pain, dysuria, frequency 9 Covert bacteriuria
and new onset incontinence increases the likelihood of a UTI in 9 Posterior midline anomaly (lumbosacral meningocele)
older children.
Ultrasound
Clinical Manifestations (varies with age) • Non-invasive
• Febrile children <5 years old (preschool-toddler) • Identify structural abnormalities
9 Fever above 38 degrees 9 Obstruction
9 Looking unwell 9 Differences in kidney size
9 Urinary symptoms (inc. frequency, urgency, dysuria) 9 Kidney damage
9 Reduced fluid intake • Post-voidal ultrasound – to check for urinary retention
• Neonates: • KUB (kidneys, ureters, urinary bladder)
9 Lethargy, Poor feeding, Jaundice 9 *First imaging study performed for a diagnosed case of UTI
9 Fever or no fever in combination of the presentation above 9 Normal kidney is vascular, same echogenicity with liver
9 UTI in neonates presents non-specific clinical symptoms and 9 Kidney whiter than liver (hyperechoic) indicates renal
sometimes with similar manifestation with sepsis* parenchymal disease

UTI: Laboratory Diagnosis DMSA Scan (dimercaptosuccinic acid)


• In identifying acute parenchymal injury following UTI
Urinary Sample Collection
9 Sensitivity of 86% | Specificity of 91%
• “The diagnosis of UTI will start with Urinalysis. It is very important
• Most sensitive investigation for acute and chronic kidney damage
not just to get any urine sample, but to get a quality urine sample.”
• Cannot differentiate damage due to UTI and congenital causes
9 Clean voided samples (common)
• Detects:
9 Catheterized sample
9 Scars after pyelonephritis
9 Suprapubic sample
9 Renal agenesis
• Dipstick testing of urine for WBC esterase and nitrite or urinary
9 Ectopic kidney
microscopy for pyuria or bacteria cannot replace urine culture
• Static renal exam; specific to the cortex
for the diagnosis of UTI.
• *Pyuria (pus / WBC in urine) – least reliable indicator for UTI
VCUG (voiding cystourethrogram)
• For high-risk patients
Sensitivity and Specificity of Components of Urinalysis
• Reference standard for identifying vesicoureteral reflex (VUR)
Test Sensitivity Specificity
• Provides information on the bladder and urethra
WBC Esterase 83% 78%
Nitrite Test 53% 98%
WBC Esterase and Nitrite 93% 72%
Microscopy: WBC >5/hpf 73% 81%
Microscopy: Bacteria 81% 83%
WBC Esterase, Nitrite, 99.8% 70%
Microscopy positive

Diagnosis Based on Urine Culture: gold standard*


• Freshly voided urine sample: >100,000 CFU/mL of a single
(gram-negative) bacterial species
• Catheter specimen: >10,000 CFU/mL
• Suprapubic aspiration: any bacterial growth Figure 538-5 (left) Dimercaptosuccinic acid renal scan showing bilateral
photopenic areas indicating acute pyelonephritis and renal scarring. LPO, left
• If the culture shows >50,000 colonies of a single pathogen posterior oblique; RPO, right posterior oblique.
(suprapubic / catheter sample), or if there are 10,000 colonies and
the child is symptomatic, the child is considered to have UTI Figure 538-6 (right) Intrarenal reflux. Voiding cystourethrogram in an infant boy
with a past history of a urinary tract infection. Note the right vesicoureteral
• In a bag sample (wee bag), if the urinalysis result is positive, the
reflux with ureteral dilation, with opacification of the renal parenchyma
patient is symptomatic, and there is a single organism cultured with a
representing intrarenal reflux.
colony count >100,000, there is a presumed UTI.

Nephrology A | 7 of 14

Controversies Surrounding Renal Imaging Sensitivity Specificity Cut-Off


• Children with VUR have 1.5x damage seen in DMSA Procalcitonin 71% 72% 0.5 ng/mL
• Half of children without VUR also have changes CRP 87% 41% 20 mg/L
• Systematic review of 13 studies showed that DMSA is a poor WBC 63% 55% 15,000/mm3
predictor of dilating VUR Even if they are said to be indicators of UTI. In reality, they cannot
• VCUG and DMSA scans are associated with significant radiation differentiate Upper from Lower unless coupled with a positive DMSA
exposure
• Lower Urinary Tract Infection: urethra, urinary bladder
Top to Bottom vs. Retrograde Studies First 9 Considered as uncomplicated infection
• National Institute of Health and Care Excellence (NICE) • Upper Urinary Tract Infection: ureter, kidneys
9 Uncomplicated first UTI of children >6 months require no 9 Considered as complicated infection
investigation
• AAP and ISPN Guidelines UTI: Treatment
9 All infants 2 -24 months with febrile UTIs should undergo • Risk of kidney damage on DMSA scan 6 -12 months after UTI
ultrasound shows no significant difference between oral antibiotics for 10-14
• All guidelines do not recommend routine VCUG & DMSA unless: days vs IV meds for 3-4 days followed by oral antibiotics vs IV
9 Ultrasound is abnormal antibiotics for 7-14 days. (Not included in the study are 1-3 months
9 Child is seriously ill old neonates)
9 Fails to respond rapidly to antibiotics • Majority of children may be treated with oral antibiotics or short
9 Recurrent infection course IV antibiotics followed by oral antibiotics

Continuous Antibiotic as Prophylaxis


• RCT’s found no difference in the efficacy of antibiotics in
preventing UTI between children with and without VUR or between
mild and severe VUR

Which Antibiotics to Use? Depends on the Organism


• on the basis of resistance patterns:
9 >50% are resistant to ampicillin
st
• Retrograde: start with VCUG first 9 30% resistant to trimethoprim and 1 gen cephalosporins
rd
9 If (+) Reflux: DMSA 9 Amoxicillin-clavulanate and to 3 gen cephalosporins are also
9 If Normal: No work up increasing due to ESBL producing E. coli
9 If (+) Hydronephrosis: MAG-3 Lasix, Renogram (MR Urogram) • Identify organism but while waiting for the result, give empirical
• Top to Bottom: Starts from a renal scan (DMSA) proceed to other treatment based on most common pathogen for the age group.
investigation depending on the result of the scan:
9 If (+) Cortical defect (pyelonephritis): proceed to VCUG Other Therapies
9 If normal: no work up done; If recurrent febrile UTI: do VCUG • Cranberry
9 If (+) Central photopenia (hydronephrosis): do sonogram 9 Inhibits the P fimbriae of E. coli preventing it from adhering to
uroepithelial cells → E. coli can be easily flushed out
Localization of UTI • Probiotics
• Standard for acute pyelonephritis is acute parenchymal damage 9 In vitro studies show bactericidal effects
on DMSA scan 9 RCTs: no significant data that proves its advantages
• Increased levels of: Procalcitonin, CRP, WBC • Vaccines
• Metanalysis of 18 studies of children with febrile UTI: 9 In adults: immunostimulants are beneficial
9 61% positive for DMSA 9 In pediatrics: no study to support
9 increase level of Procalcitonin, CRP, WBC count

Table 538-3 Guideline Recommendations for Diagnostic Evaluation Following a Febrile Urinary Tract Infection in Infants
GUIDELINE ULTRASONOGRAPHY VCUG LATE DMSA SCAN
National Institute for Health and (see Table 538-4)
Care Excellence (NICE)*
American Academy of Pediatrics Yes If abnormal ultrasonogram No
Italian Society for Paediatric Yes If abnormal ultrasonogram or if risk If abnormal ultrasonogram or VUR
Nephrology (ISPN) factors are present†
* Upper urinary tract dilation on ultrasonography, poor urinary flow, infection with organism other than E. coli, or family history of vesicoureteral reflux.
† Abnormal antenatal ultrasonogram of fetal urinary tract, family history of reflux, septicemia, renal failure, age younger than 6 mo in a male infant, likely family
noncompliance, incomplete bladder emptying, no clinical response to appropriate antibiotic therapy within 72 hr, or infection with organism other than E. coli.

Table 538-3 Guideline Recommendations for Diagnostic Evaluation Following a Febrile Urinary Tract Infection in Infants
Type of Infection
CHILD AGE AND TESTS RESPONDS WELL TO TX WITHIN 48 HR ATYPICAL INFECTION RECURRENT INFECTION
CHILDREN YOUNGER THAN 6 MO OLD
Ultrasound scan during acute infection No Yes Yes
Ultrasound scan within 6 wk of infection Yes No No
DMSA scan 4-6 mo after acute infection No Yes Yes
Micturating cystograms Consider if ultrasound scan abnormal Yes Yes
CHILDREN 6 MO-3 YR OLD
Ultrasound scan during acute infection No Yes No
Ultrasound scan within 6 wk of infection No No Yes
DMSA scan 4-6 mo after acute infection No Yes Yes
Micturating cystograms No Not routine, consider if dilation on ultrasound, poor urine flow, non–E. coli
infection, or family history of vesicoureteric reflux
CHILDREN OLDER THAN AGE 3 YR
Ultrasound scan during acute infection No Yes No
Ultrasound scan within 6 wk of infection No No Yes
DMSA scan 4-6 mo after acute infection No Yes Yes
Micturating cystograms No No No

Nephrology A | 8 of 14

V. ISOLATED GLOMERULAR DISEASES WITH • Prophylactic antibiotics and tonsillectomy have no proven benefit.
• Renal transplantation (no absolute cure) or dialysis
RECURRENT HEMATURIA {📖 510} • Allograft loss caused by IgA nephropathy in 15-30% of patients
1. IgA Nephropathy (Berger) {📖 510.1} causes recurrence.
• Most common chronic glomerular disease worldwide
• Benign hematuria without systemic manifestations
2. Alport Syndrome {📖 510.2}
• Characterized by a predominance of IgA within mesangial • Aka hereditary nephritis
deposits of the glomerulus • Genetically heterogeneous disease caused by mutations in the
• Initially presents with significant microscopic hematuria without genes coding for type IV collagen, a major component of
HTN or edema. basement membranes
9 HTN and edema occurs only as disease progresses. • Clinical Manifestations:
• Natural history: 9 Asymptomatic microscopic hematuria
9 Initial and persistent microscopic hematuria, then 9 Single or recurrent episodes of gross hematuria commonly
9 Gross hematuria, then
occurring 1-2 days after an upper respiratory infection (50%)
9 Eventually disappears, with URTI
9 Proteinuria in boys; may be absent, mild, intermittent in girls.
• An immune complex disease caused by abnormalities in IgA 9 Progressive proteinuria
• Familial clustering =? genetic factors
9 Bilateral sensorineural hearing loss
• Genome-wide linkage analysis: 6q22-23
9 Ocular abnormalities: anterior lenticonus (extrusion of the
• Other diseases with prominent IgA mesangial deposition
9 Rheumatic Arthritis (RA)
central portion of the lens into the anterior chamber), macular
9 Ankylosing Spondylitis (AS)
flecks, and corneal erosions.
9 Reiter syndrome (arthritis, urethritis, bilateral conjunctivitis) 9 Leiomyomatosis of the esophagus, tracheobronchial tree, and

9 Hepatic cirrhosis female genitals in association with platelet abnormalities: rare

IgA N: Clinical and Laboratory Manifestations 3. Thin Basement Membrane Disease {📖 510.2}
• Seen more often in male than in female patients. • Presence of persistent microscopic hematuria and isolated
• Westerners present with gross hematuria; Asians (Japan) thinning of the GBM (and, occasionally, tubular basement
present with microscopic hematuria and/or proteinuria membranes) on electron microscopy.
• Other types of Presentation: • Microscopic hematuria is often initially observed during child-
9 Acute Nephritic; Acute Nephrotic; Combined Nephritic- hood and may be intermittent.
Nephrotic syndrome (sig. proteinuria and hematuria) • Episodic gross hematuria can also be present, particularly after a
• Gross hematuria may occur in association with URTI or GI
respiratory illness.
infection, and may be associated with loin pain.
• Benign Familial Hematuria
9 Often occurs within 1-2 days of onset of the infection, in
9 Isolated hematuria in multiple family members without renal
contrast to the longer latency period of PSGN
dysfunction
• IgA nephropathy: Gross hematuria may recur.
9 Although most of these patients will not undergo renal biopsy,
9 PSGN: Gross hematuria does not recur.
it is often presumed that the underlying pathology is TBMD.
• Proteinuria is frequently in the nephritic range (<1000 mg/24hr) in
patients with asymptomatic, microscopic hematuria. • Heterozygous mutations in the COL4A3 and COL4A4 genes,
• Mild to moderate hypertension which encode the α3 and α4 chains of type IV collagen present in
9 Most often seen in patients with nephritic/nephrotic syndrome, the GBM, result in TBMD.
but rarely severe enough to result in hypertensive emergencies • Rare cases of TBMD progress, and such patients develop
• IgA Nephropathy: Normal serum C3 significant proteinuria, hypertension, or renal insufficiency.
9 PSGN: Decreased C3
• Serum IgA have no diagnostic value VI. HSP (Henoch-Schonlein Purpura) NEPHRITIS {📖 515}
9 Only elevated in 15% of patients.
• IgA Nephropathy with systemic manifestations
• Progressive disease develops in 20-30% of children at 15-20 years
• Also referred to as Anaphylactoid Purpura
after onset. • Small vessel vasculitis characterized by a tetrad of*
1. Purpuric rash
IgA N: Prognosis § Palpable purpura, symmetrical, in gravity-dependent areas
Good • Isolated hematuria (lower extremities) or pressure points (buttocks)
• Isolated proteinuria § Greater than macule; >1cm; elevated
Poor • Persistent hypertension § Red, becomes purplish, to yellow-brown, then disappears
• Diminished renal function 2. Arthritis
• Heavy or prolonged proteinuria § Big joints (wrist, elbow, knees, ankle)
• Combination
§ Signs of inflammation (rubor, calor, dolor, tumor, functio laesa)
Worse • Diffuse mesangial proliferation
§ DDx: SLE/ RA/ Septic arthritis/ JRA (small joints, primarily)
(>5-8 mesangial cells affected)
3. Abdominal pain
• Extensive glomerular crescents (also seen in RPGN)
§ Primary peritonitis
• Glomerulosclerosis
§ Can present as an acute abdomen; mistaken for appendicitis
• Tubulointerstitial changes – inflammation, fibrosis
(chronic indicator) 4. Glomerulonephritis

IgA N: Treatment – No Drug of Choice HSP Nephritis and IgA Nephropathy have identical findings,
• Proper BP control
except that systemic findings are only found in HSP.
• Fish oil – with anti-inflammatory omega-3 FA
9 Decrease rate of renal progression HSP N: Pathogenesis
• Immunosuppressive therapy with corticosteroids or more • Remains unknown
intensive multidrug regimens (i.e. cyclophosphamide) may be • Appear to be mediated by the formation of immune complexes
beneficial in some patients containing polymeric IgA1 within capillaries of the skin, intestines,
• ACEI and ARBS and glomerulus
9 Still under study concerning reduction of proteinuria and
retarding renal progression

Nephrology A | 9 of 14

HSP N: Diagnosis RVT: Clinical Manifestations


• Non-specific: based on clinical findings • The development of RVT is classically heralded by:
9 Gross hematuria: 20-30% 9 Sudden onset of gross hematuria
9 Isolated microscopic hematuria 9 Unilateral or bilateral flank masses
9 Hematuria and proteinuria • However, patients can also present with any combination of:
9 Acute nephritic signs and symptoms 9 Microscopic hematuria
9 Nephrotic signs and symptoms 9 Flank pain
9 Renal insufficiency 9 Hypertension
• Specific 9 Microangiopathic hemolytic anemia with thrombocytopenia
9 Renal manifestations occurring up to *12 weeks after initial 9 Oliguria
presentation • RVT is usually unilateral
9 Bilateral RVT results in acute renal failure
Ureteritis
• Uncommon urologic manifestation RVT: Diagnosis
• Associated with loin pain and renal colic • Suggested by the development of hematuria and flank masses in
• Usually seen in children <5 years patients seen in the high-risk clinical settings or with the
• Frequently leads to ureteral stenosis and strictures, causing predisposing clinical features noted above.
hydronephrosis that requires surgical correction • CBC: microangiopathic hemolytic anemia, thrombocytopenia
• Ultrasonography: marked renal enlargement
HSP N: Treatment • Radionuclide studies: reveal little or no renal function in the
• There are no controlled data demonstrating that steroids, cytotoxic affected kidney(s)
agents, or anticoagulants alter the course of HSP nephritis. • Doppler flow studies of IVC and renal vein confirm the diagnosis.
• Uncontrolled studies suggest: • Contrast studies: should be avoided to minimize the risk of
9 Potential value of high-dose steroids, tacrolimus, cytotoxic further vascular damage.
therapy (cyclophosphamide, azathioprine) in patients with
crescentic GN or significant proteinuria. RVT: Differential Diagnosis
9 Additional dipyridamole and/or heparin/warfarin may provide • Includes other causes of hematuria that are associated with rapid
added benefit. development of microangiopathic hemolytic anemia or
enlargement of the kidney(s).
HSP N: Prognosis 9 Hemolytic uremic syndrome
• Generally favorable 9 Hydronephrosis
• Signs & symptoms may continue for several months, especially GN 9 Polycystic kidney disease
• Risk of chronic renal insufficiency is 2-5% 9 Wilms tumor
• Presentation with isolated microscopic hematuria alone carries 9 Renal abscess
the best prognosis 9 Hematoma
• Acute nephritic syndrome and Nephrotic syndrome carries the • All patients should be evaluated for congenital and acquired
highest risk of developing chronic renal failure. hypercoagulable states.

VII. RENAL VENOUS THROMBOSIS {📖 519.7} RVT: Treatment


• Aggressive supportive intensive care
RVT: Epidemiology
9 Correction of fluid and electrolyte imbalance
Renal vein thrombosis (RVT) occurs in 2 distinct clinical settings: 9 Treatment of renal insufficiency
1. In newborns and infants*, RVT is commonly associated with: • Anticoagulation or thrombolysis remain controversial
9 Asphyxia 9 Streptokinase
9 Dehydration 9 Urokinase
9 Shock 9 Recombinant t-PA
9 Sepsis 9 Difficult in patients with sepsis and deranged coagulation
9 Congenital hypercoagulable states • Surgical thrombectomy – indicated for thrombosis of the IVC
9 Maternal diabetes • Nephrectomy – for children with severe hypertension secondary
2. In older children*, RVT is seen in patients with: to RVT who are refractory to antihypertensive medications
9 Nephrotic syndrome
9 Cyanotic heart disease RVT: Prognosis
9 Inherited hypercoagulable states
• Perinatal mortality from RVT has decreased significantly due to
9 Sepsis
intensive support.
9 Following kidney transplantation
• Partial or complete renal atrophy is a common sequela of RVT in
9 Following exposure to angiographic contrast agents
the neonate, leading to an increased risk of renal insufficiency,
renal tubular dysfunction, and systemic hypertension.
RVT: Pathogenesis
• These complications are also seen in older children.
• RVT begins in the intrarenal venous circulation and can then • However, recovery of renal function is common in older children
extend to the main renal vein and even the inferior vena cava. with RVT resulting from nephrotic syndrome or cyanotic heart
• Thrombus formation is mediated by endothelial cell injury disease with correction of the underlying etiology.
resulting from hypoxia, endotoxin, or contrast media. • RVT during neonatal period requires follow-up for life because of
• Other contributing factors include: early onset hypertension.
9 Hypercoagulability
§ Nephrotic syndrome, factor V Leiden deficiency
9 Hypovolemia and decreased venous blood flow
§ Septic shock, dehydration, nephrotic syndrome
9 Intravascular sludging
§ Polycythemia

Nephrology A | 10 of 14

VIII. WILMS TUMOR/NEPHROBLASTOMA {📖 499} WT: Treatment


• Surgery
• Complex mixed embryonal neoplasm of the kidney
• Patency of the IVC should be established prior to resection
• Composed of 3 elements:
• If IVC not patent: chemotherapy should be administered
9 Blastema
9 Epithelia
9 Stroma
Chemotherapy Guidelines
S1, S2 with favorable • Vincristine
• 8 cases/million children <15 years of age
histology • Dactinomycin
• Peak: 2-5 years of age S3 with • Radiation in tumor
• Second most common malignant abdominal tumor in childhood favorable histology bed
• May arise in one or both kidneys • Radiation to sites of
9 One side may be bigger than the other
S4 with
• Vincristine known disease,
favorable histology
• The incidence of a bilateral Wilms is 7%. • Dactinomycin particularly the lungs
• May be associated with the following: • Doxorubicin • Surgical resection
9 Hemihypertrophy – one side is bigger, very obvious S4 with may be considered,
9 Aniridia – absence of iris, unilateral or bilateral tumor in the liver as opposed to
radiation
9 Other congenital anomalies, usually of GUT
Genitourinary Anomalies – most commonly associated S4, resistant tumors
• Consider surgical resection and alternate or
with chemo or
§ Hypoplasia (small kidney, few glomeruli) investigational chemotherapy and alternate or
radiation failure;
§ Fusion (horseshoe kidney) investigational chemotherapy
recurring tumors
§ Ectopia (not in renal fossa), 2 kidneys on the same side
• Radiation to tumor
§ Duplication of the collecting systems (hydronephrosis at • Vincristine bed and sites of
the upper portion of the system) Unfavorable • Dactinomycin established
§ Hypospadias histology • Doxorubicin metastasis
ª Mild: meatus displaced at apex • Cyclophosphamide • More aggressive
ª Moderate: at the middle treatment
ª Severe: at the perianal area
• Chemotherapy is administered
§ Cryptorchidism Wilms tumor
• Diagnosis established by percutaneous needle
9 Variety of syndromes
not operable
biopsy
§ WAGR syndrome
§ Denys-Drash syndrome • Chemo identical to
§ Beckwith-Wiedemann syndrome
Bilateral Wilms that employed in • Surgical extirpation
tumor inoperable tumors after chemotherapy
§ Pearlman syndrome
utilized
§ Sotos syndrome
§ Neurofibromatosis (von Recklinghausen disease)
§ von Willebrand disease
Chemotherapy Guidelines
• Chemotherapy is administered for all Wilms tumors that appear
• Majority are sporadic - 1-2% are familial, AD
• Familial cases are associated with diagnosis at an earlier age inoperable.
• Diagnosis is established by percutaneous needle biopsy.
• Increased frequency of bilateral disease
• Chemotherapy for bilateral Wilms tumor is identical to that
• Congenital absences are usually absent
• Not associated with Wilms tumor gene (WT1) employed for inoperable tumors and is utilized to render the tumor
amenable to surgical extirpation.
WT: Clinical Manifestations
Table 499-4 Staging of Wilms Tumor
• Abdominal mass - approached as malignant mass Stage I Tumor confined to the kidney and completely resected.
9 Smooth, firm Renal capsule or sinus vessels not involved. Tumor not
9 Occasionally cross the midline ruptured or biopsied. Regional LN examined and negative.
• Abdominal pain, vomiting Stage II Tumor extends beyond the kidney but is completely resected
• Hematuria with negative margins and lymph nodes. At least 1 of the
following has occurred: (a) penetration of renal capsule,
• Hypertension
(b) invasion of renal sinus vessels.
Histology Stage III Residual tumor present following surgery confined to the
Favorable Unfavorable abdomen, including gross or microscopic tumor; spillage
of tumor preoperatively or intraoperatively; biopsy prior to
• Conventional form • Markedly large nuclei
nephrectomy, regional LN metastases; tumor implants on
characterized by blastema, • Hyperchromatism and multipolar the peritoneal surface; extension of tumor thrombus into
epithelia, stromal elements figures of nuclei the IVC including thoracic vena cava and heart.
• Devoid of ectopia, anaplasia • Areas of anaplasia may be focal Stage IV Hematogenous metastases (lung, liver, bone, brain, etc.) or
or diffuse; predict higher rates of LN metastases outside the abdominopelvic region.
tumor relapse and death Stage V Bilateral renal involvement by tumor.
• Clear cell sarcoma is a subtype
of the unfavorable form and WT: Prognostic Factor
usually metastasizes to the bone
• Major prognostic factors are tumor size, stage and histology
• Prognosis is worse with:
WT: Diagnosis
9 Large tumor (>500 g)
• Abdominal mass: urgent nature of the condition
9 Advanced stage (S3, S4)
• Complete PE, CBC
9 Unfavorable histological type
• Liver and Kidney FT’s
• Wilms tumor constitutes a paradigm of successful multidisciplinary
• Specific tumor markers
treatment; >60% of patients with all stages generally survive.
• Flat plate of the abdomen
• Stages I-III have a cure rate varying from 88-98%.
• Ultrasonography: often used
• CT/MRI
• Chest X-ray: detection of metastasis, useful for pre-op

Nephrology A | 11 of 14

IX. READING ASSIGNMENTS SLE GN: Treatment


• Pediatric specialist required
1. SLE Nephritis {📖 514} • Immunosuppressive tx → establish clinical & serologic remission
• Characterized by: 9 Normalization of anti-DNA, C3 and C4 levels

9 Fever, weight loss, rash, hematologic abnormalities, arthritis,


• Initial: Prednisone (1-2 mg/kg/day) in 2-3 divided doses
9 Tapered over 4-6 months, 4-6 week after remission
and involvement in the heart, lungs CNS and kidneys
• Majority are adolescent females • More severe forms (III, IV) – 6 months IV Cyclophosphamide at
500 -1000 mg/m2
9 Followed by every 3 months for 18 months
WHO Classification
• Class I, II – steroid sparing agent Azathioprine (1.5-2.0 mg/kg OD)
Class I Nephritis No histologic abnormality
Class II Nephritis Glomeruli with mesangial deposits • Mild – Mycophenolate mofetil
(mesangial lupus nephritis) with Ig & Complement • Refractory Class IV – Rituximab, a chimeric monoclonal Ab
Class IIA Mild mesangial hypercellularity specific for human CD20
Class IIB Moderate mesangial hypercellularity
and ↑ matrix SLE GN: Prognosis
Class III Nephritis Almost all glomeruli • Aggressive immunotherapy → improved prognosis
(focal segmental lupus GN) • Mesangial deposits • Renal survival without dialysis in 80% after 10 years
• Subendothelial deposits between • Highest risk for ESRD – diffuse proliferative class IV
endothelial cells and glomerular • Risks of treatment
basement membrane 9 Chronic immunosuppressive treatment
Occasional 9 Chronic steroid treatment: osteoporosis, obesity, HPN, DN
• Capillary wall necrosis, crescent
9 Cyclophosphamide (>20g): high risk of malignancy of infertility
formation, and sclerosis
Class IV Nephritis All glomeruli
(diffuse proliferative lupus • Significant mesangial deposits 2. Hemolytic Uremic Syndrome {📖 518}
nephritis) • Subendothelial deposits of Ig • Most common cause of acute RF in young children
most common and most and complement • Classic triad
severe • Mesangial proliferation 9 Microangiopathic hemolytic anemia
Frequently 9 Thrombocytopenia
• Capillary walls thickened,
9 Uremia
secondary to subendothelial
• Features common to TTP except latter (TTP) tends to occur in
deposits (wire loop lesion) often
necrosis, crescent formation, and young adult women as relapsing illness with fever, serious CNS
scarring involvement and thrombocytopenia
Class V Nephritis Resembles idiopathic membranous
(membranous lupus nephritis) glomerulopathy, except for presence HUS: Etiology
least common of mild to moderate mesangial Typical HUS • Acute enteritis with diarrhea caused by Shiga-like
proliferation (post-diarrheal) verotoxin-producing E. coli O157:H7 – 80%
• Shigella and less commonly:
SLE GN: Pathology 9 Salmonella, campylobacter, Streptococcus
pneumonia, Bartonella
• Clinical manifestations are mediated by immune complexes
9 Coxsackie, echovirus, influenza, varicella,
• Aberrations in both B- and T-cell function
HIV, EBV
• WHO classification based on light microscopy +
Atypical HUS • UTI due to Shiga toxin-producing E. Coli
immunofluorescence + electron microscopy features • Post-infectious outside GIT (Strep pneumoniae)
9 Transformation from one class to another is common
• Oral contraceptives, mitomycine or cyclosporine,
9 Likely to occur in inadequately treated patients ganciclovir, crack cocaine, quinine
• Exposure to pyran copolymer
SLE GN: Clinical Manifestations • Familial occurrence – no diarrhea, autosomal
• Clinical evidence in 30 -70% of children recessive or autosomal dominant
9 Kidney disease is one of the most common features of SLE in 9 Mutation in complement component H
childhood, may even be the only one 9 Complete deficiency of von Willebrand factor
• Milder forms (all class II, some class III) metalloprotease and membrane cofactor
9 Hematuria
protein
• Rare association:
9 Normal renal function
9 SLE, malignant hypertension, preeclampsia,
9 Proteinuria of <1 g/24h
post-partum renal failure, radiation nephritis
• Some class II, all class IV
9 Hematuria and proteinuria
HUS: Pathology
9 Reduced renal function
9 Nephrotic syndrome • Initial glomerular changes due to subendothelial and mesangial
9 Acute renal failure deposition of granular, amorphous, material of unknown origin
• Class IV can rarely have normal urinalysis 9 Thickening of capillary walls
• Class V: Nephrotic Syndrome 9 Narrowing of capillary lumens
9 Widening of mesangium
SLE GN: Diagnosis • Fibrin thrombi in glomerular capillaries/ arterioles → cortical
necrosis
• Circulating ANAs – suggestive
• Severe – partial or total sclerosis, ischemia, concentric intimal
• Ab to native dsDNA – confirmatory
proliferation → vascular occlusion
• C3, C4 depressed
• Renal biopsy if no clear correlation between clinical manifestations
HUS: Pathogenesis
and severity of renal involvement
• Results guide immunosuppressive treatment • Endothelial cell injury
• Localized clotting in the kidney capillaries/ arterioles
9 DIC is unusual; Low C3 in non-diarrheal forms
• RBC damage → Microangiopathic anemia
• Intrarenal and diffuse microvascular platelet adhesion/ damage →
Thrombocytopenia

Nephrology A | 12 of 14

HUS: Clinical Manifestations 3. Tubulointerstitial Nephritis {📖 532}


• Most common in children <4 years old
• Characterized by tubulointerstitial inflammation and damage with
• Onset usually preceded by gastroenteritis characterized by
relative sparing of glomeruli and vessels
fever, vomiting, abdominal pain, and diarrhea (watery → bloody) • Acute and chronic forms; Primary or Systemic/Secondary
9 Less common: URTI
• 5-10 days later: sudden onset of pallor, irritability, weakness,
lethargy, oliguria
3.1 Acute TIN
• PE: dehydration, edema, petechiae, hepatosplenomegaly, and • Hallmarks: lymphocytic infiltration of tubulointerstitium, tubular
marked irritability edema, varying degrees of tubular damage
• Eosinophils (drug -induced), granulomas T cell-mediated immune
HUS: Diagnosis and Differentials mechanism, unknown pathology
• Clinical findings – always consider HUS in pediatric ARF
A-TIN: Clinical Manifestations
• Differentials:
• Classic: Fever, Rash, arthralgia + increased serum creatinine
9 SLE and malignant hypertension
9 Rash: vary from maculopapular to urticarial, often transient
9 Bilateral RVT (differentiate by Doppler)
• Non-specific: nausea, vomiting, fatigue, weight loss,
Complete Blood Count (CBC) flank pain (2° to stretched renal capsule); 30-40% non-oliguric
• Hgb: 5-9 g/dL; ↑ plasma Hgb, ↓ plasma haptoglobin • Microscopic hematuria: Significant Hematuria or Proteinuria is
• PBS: helmet cells, burr cells, fragmented RBCs uncommon – except with NSAIDs
• Reticulocyte: moderately elevated • Urinalysis: WBC casts, granular / hyaline casts, no RBC casts
• (-) Coombs test • Eosinophils - not sensitive or specific
• Significant leukocytosis: usually > 3-,000/ mm3
• Thrombocytopenia (90%): 20,000 100,000/ mm3 A-TIN: Diagnosis
• Normal PTT and PT (no vitamin K deficiency) • Clinical and laboratory
• Careful history of timing especially with respect to drug exposure
Acute Renal Injury (usually 1-2 weeks after)
• Varies: mild RI to acute oliguric or anuric RF requiring dialysis
• RUS: not diagnostic but may see echogenic, enlarged kidneys
9 Elevated creatinine level
• Removal of an agent with improvement: highly suggestive
• Urinalysis: surprisingly mild • Severe, rapidly deteriorating – do biopsy – establishes Dx
9 Low-grade microscopic hematuria and proteinuria

Imaging A-TIN: Treatment and Prognosis


• Barium contrast studies: • Supportive, address complications of ARF
9 colonic spasms, transient early filling defects 9 Hyperkalemia or volume overload
• Radio: colitis “thumb-printing” • Corticosteroids
• Best prognosis: rapid improvement without treatment
Case Classification (CDC) • Guarded prognosis: prolonged renal insufficiency
• Confirmed
9 Acute illness diagnosis as HUS or TTP meeting lab criteria with 3.2 Chronic TIN
history of acute or bloody diarrhea in past 3 weeks
• Probable
• In children, commonly due to underlying congenital renal disease:
9 Meeting laboratory criteria with no clear history of acute or
obstructive uropathy or vesicoureteral reflux or antimicrobials
• Idiopathic: more common in adults
bloody diarrhea in past 3 weeks or
• Seen in all forms of progressive renal disease
9 Onset within 3 weeks after acute or bloody diarrhea and meets
• Severity is most important factor in progression to ESRD
the laboratory criteria except that microangiopathic changes
are not confirmed Undefined pathophysiology, maybe immune mediated

JN-MCKD Complex
HUS: Complications
• Group of inherited cystic renal diseases that share common
• Anemia, acidosis, hyperkalemia, fluid overload, HF, HTN, uremia histologic phenotype of chronic TIN
• Extra-renal (life-threatening) • Juvenile Nephrophthisis (JN)
9 CNS: irritability, seizures, infarcts of BG and CC, cortical
9 Rare; AR (however, in Europe cause 10-2% of ESRD)
blindness, coma 9 Polyuria, growth failure, “unexplained” anemia and CRF in late
9 GIT: Ischemic or inflammatory colitis, intestinal perforation, childhood or adolescence
intussusception and hepatitis 9 Variants
9 Pancreas: Focal necrosis → acute pancreatitis, glucose
§ Senior-Loken syndrome (retinitis pigmentosa)
intolerance, insulin – dependent DM, High lipase § Joubert syndrome
9 Heart: Pericarditis, myocardial dysfunction, arrhythmias § Oculomotor apraxia type Cogan
9 Other: Skin necrosis, parotitis, adrenal dysfunction, • Medullary Cystic Kidney Disease (MCKD)
rhabdomyolysis 9 AD; typically presents in adulthood

HUS: Prognosis and Treatment Tubulointerstitial Nephritis with Uveitis


• Supportive care • Rare autoimmune syndrome
9 Fluid and electrolytes: Aggressive nutrition • Chronic TIN + anterior uveitis + bone marrow granulomas
9 Control Hypertension: Early dialysis • Usually adolescent females
• Avoid antibiotics (Higher HUS risk)
• Anti-thrombotic treatment, Plasmapheresis: no proven benefit C-TIN: Pathology
• Diarrheal – 90% survive: 12% ESRD or death • Grossly: Kidneys – pale and small for age
• Worst Prognosis: • Micro: tubular atrophy and “drop out” with interstitial fibrosis and a
9 CNS symptoms patchy lymphocytic interstitial inflammation
9 WBC: >20,000, ischemic colitis, hypertension • JN: characteristic small cysts in corticomedullary region
• Transplant does not guarantee no recurrence • Primary CTIN: glomeruli relatively spared until late disease
• F-U required: some complications manifest 20 years later

Nephrology A | 13 of 14

C-TIN: Clinical Manifestations 6. Henoch-Schonlein Purpura Nephritis may manifest after the
• Often non-specific, may have s/sx of chronic renal insufficiency diagnosis of HSP as late as:
• Fatigue, growth failure, polyuria, polydipsia, and enuresis A. 2 weeks C. 8 weeks
• Anemia is common (JN) B. 4 weeks D. 12 weeks
• Significant hypertension: tubular damage = salt wasting 7. A 7-month-old male infant with generalized edema is being worked up
for nephrotic syndrome. Since a 24-hour urine collection is difficult to
C-TIN: Diagnosis do in this age group, you would request for the following instead:
• Signs and Symptoms of renal tubular damage: polyuria, A. Urine dipstick test for albumin C. Urine protein/creatinine ratio
increase creatinine + history suggesting chronic disease (long - B. Urine albumin/calcium ratio D. Urine protein chromatography
standing enuresis or anemia)
8. 5 days PTC, a 6-year-old male had facial edema, distention of the
• RUS: evidence of chronicity, corticomedullary cysts (JN) or
abdomen and edema of lower ext. This was associated with tea-
obstructive uropathy
colored urine, oliguria, headache, and vomiting. On further history, it
9 Vesicocystourethrogram: VUR or bladder abnormality
was noted that he sustained a wound at the left foot that became
• JN: molecular diagnosis
infected 14 days PTC. Urinalysis showed significant hematuria and
• Unclear: do biopsy if not too advanced proteinuria with a low C3 and +ASO titer. Start with:
A. Penicillin C. Digoxin
C-TIN: Treatment and Prognosis B. Prednisone D. Omeprazole
• Fluids and electrolytes; Avoid nephrotoxic agents
• Obstructive uropathy: salt supplement, K+ binding resin 9. What is the latent period on the case on #8?
(Kayexalate → watch out for arrhythmia) A. 7 days C. 11 days
B. 9 days D. 14 days
• Antibiotic prophylaxis
• Variable prognosis, ~ESRD (*JN) 10. For the treatment of Nephrotic syndrome, what is the duration of the
9 Patients with obstructive uropathy or vesicoureteral reflux can 2
initial dose of 60 mg/m /day of prednisone?
have a variable degree of renal damage and thus a variable A. 6 weeks (proposed answer) C. 10 weeks (answer key)
course. ESRD can develop over months to years. Patients with B. 8 weeks D. 12 weeks
JN uniformly progress to ESRD by adolescence. 11. Other than treating UTI in children with appropriate antibiotics, which of
the following is an important part of history which, if present, will need
X. COMPILED SAMPLEX to be addressed?
A. Bladder and bowel dysfunction of the child
Shifting – Identification / Enumeration: B. Dietary and fluid intake
1. Serologic test to assess previous streptococcal infection? C. Past illnesses
9 ASO titer, Neuraminidase test, DNAse test D. Environmental and sanitation conditions
2. Initial imaging procedure for recurrent UTI?
9 KUB ultrasound 12. A 7-year-old male was complaining of dysuria. The patient also had a
3. DOC in IGA Nephropathy? history of a recent cough and colds but no fever. He had gross
9 None; there is no DOC for IGA Nephropathy hematuria a day prior to consult characterized as bright red urine
4. Manifestations of HSP Nephritis except for purpuric rash? with blood clots. PE and vital signs were normal. Initial consideration:
9 Abdominal pain, Arthritis, Glomerulonephritis A. Glomerulonephritis C. Blood disease
5. Complication of Nephrotic Syndrome? B. Hemorrhagic Cystitis D. Intravascular hemolysis
9 Thrombosis, Hypovolemia, Hyperlipidemia, Infection (peritonitis)
13. Hematuria is likely glomerular in nature if:
6. Etiology of PSGN?
A. It is massive
9 Immune complex formation
B. The color of urine is tea-colored
7. Latent period of PSGN?
C. Urinalysis shows significant hematuria and proteinuria
9 10-14 days
D. Urinalysis has a fixed specific gravity suggesting inability of the
8. When can you give live vaccine in Nephrotic syndrome?
kidneys to dilute or concentrate the urine
9 Can be given 4 weeks after cessation of treatment
9. # of RBC to be considered as Significant Hematuria? 14. The best way to document significant protein in the urine is:
9 >5 RBC/hpf on >2 occasions A. Urine dipstick test for albumin
10. Least reliable indicator for UTI? B. 24-hour urine protein/albumin concentration
9 Pyuria (Pus cells / WBC in the urine) C. Urine protein chromatography
D. Urine protein/creatinine ratio
Compiled Major Exam Questions:
1. This renal malformation is associated with Wilms tumor? 15. The most common clinical presentation of UTI in neonates is ★
A. Duplicating renal collecting system A. Urgency C. Fever
B. Neurogenic bladder B. Flank mass D. Sepsis syndrome
C. Vesicoureteral reflex
16. The following is an important renal imaging study that must be done for
D. Solitary renal cyst
patients diagnosed with UTI:
2. Main pathology leading to manifestations of Nephrotic syndrome A. Plain film of the abdomen C. IVP
A. Hypercholesterolemia B. KUB Ultrasound D. CT Scan
B. Massive glomerular protein loss in the urine
17. The average period between the initial streptococcal infection and the
C. Tubular mal-reabsorption of protein
nephritic signs and symptoms is:
D. Elevated levels of renin and aldosterone
A. 1 week C. 3 weeks (answer key)
3. The clinical findings of abdominal pain, arthritis, and purpuric rash B. 2 weeks (proposed answer) D. 4 weeks
on the lower extremities are suggestive of: 18. A 5-month-old male infant was brought for consult because of high-
A. Membranous GN C. Potter’s syndrome
grade fever for 12 hours duration. The child has been irritable since the
B. SLE Nephritis D. Henoch-Schonlein purpura
start of fever. Urinalysis was suggestive of UTI and you performed a
4. Neonates are at risk for RVT if this condition is present. suprapubic tap with results of Klebsiella sp. with a colony count of
A. Nephrotic Syndrome C. Dehydration 10,000 CFU/mL. What will you do?
B. Cyanotic Heart Disease D. Use of contrast media A. Repeat urinalysis and urine culture since colony count is low
B. Re-assure mother that fever would subside eventually since
5. The following is true of post-infectious glomerulonephritis: culture results are normal
A. Antibiotic treatment given early in streptococcal infections modifies C. Start patient on antibiotics against Klebsiella since urine culture is
the clinical course of PIGN. conclusive of UTI
B. Most viral causes have a long latent period. D. Do other lab test to look for other source of infection
C. A complicated course is anticipated for viral PIGN.
D. The prognosis of acute PSGN is guarded. 📌 No proofreading done. Use at your own risk.

Nephrology A | 14 of 14
PEDIATRICS : HYPERTENSION, UTI, WILMS TUMOR
Irish Senen B. Chang-Arellano, MD, DPPS, DPSN, PNSP FEU-NRMF Batch 2022

HYPERTENSION

▪ The preferred method is by auscultation using a


sphygmomanometer cuff appropriate for the size of the child’s arm
▪ Simplified blood pressure classification in adolescents 13 years of
age and older that aligns with forthcoming American Heart TECHNIQUE
Association/American College of Cardiology adult blood pressure ▪ Sphygmomanometer must be standardized, calibrated and cleaned
guidelines regularly
SYSTEMIC HYPERTENSION ▪ Inflatable bladder should cover at least 2/3 of the length of the arm
▪ Child must be relaxed and reassured, at least 5 minutes
PRIMARY (ESSENTIAL) HYPERTENSION
▪ Bladder must be inflated 30 mmHg beyond the disappearance of
▪ Common in adults
the palpable pulse.
▪ Major risk factor for myocardial infarction, cerebrovascular accident, ▪ Speed of inflation should not exceed 2mm Hg/sec.
and renal failure ▪ There should be at least a 2 min rest between measurements, with
PEDIATRIC POPULATION the cuff completely deflated
▪ Asymptomatic although may manifest target organ damage ▪ Manometer must be at the level of the heart.
▪ 40% have left ventricular hypertrophy ▪ The right arm is preferred in repeated measures of BP.
▪ Increased carotid intima-to-intima thickness (early atherosclerosis) ▪ Korotkoff sounds
ELEVATED BP o The appearance of the 1st Korotkoff sound is taken to be the
▪ Children with BP >90th percentile exhibit a 2.4-fold greater risk of systolic BP
having hypertension as adults o The point of muffling and the point of disappearance is often used
▪ Almost half of hypertensive adults had a BP>90th percentile as as the diastolic BP reference point
children Arm circumference
▪ Adolescent hypertension is also an independent predictor of both should be measured
end-stage renal disease and left ventricular dysfunction in middle- at midline
aged men. Blood pressure cuff
▪ Based on 90thpercentile BP values for children at 5thheight showing size
percentile estimation based on
▪ Negative predictive value arm circumference
>99%
INFANTS AND YOUNG
CHILDREN Recommended Dimensions for BP Cuff Bladders
<1% have underlying disease Age Range Width (Cm) Length (Cm) Max Arm
Severe symptomatic Circ (Cm)
hypertension in children is Newborn 4 8 10
usually caused by secondary Infant 6 12 15
hypertension.
Child 9 18 22
OLDER SCHOOL-AGE CHILDREN Small Adult 10 24 26
AND ADOLESCENTS Adult 13 30 34
▪ 9 % have prehypertension Large Adult 16 38 44
▪ 3-4% have hypertension Thigh 20 42 52
▪ Calculated so that the largest arm would still allow the bladder to
MEASUREMENT OF BP IN CHILDREN encircle arm by at least 80%
Example: 3y/o 99cm BP112/84
Normal Circumstances
• Children >3 years old who are seen in a medical setting should
have their BP measured at least once during every health care
episode. Children < 3years old should have their BP measured in
special circumstances
Special Circumstances
• Hx of prematurity, VLBW or other NICU care sit
If initial BP is elevated
• CHD [repaired or non-repaired]
▪ Perform 2 additional oscillometric or auscultatory BP
• Recurrent UTI, hematuria or proteinuria measurements
• Known renal disease or urologic malformations ▪ Discard 1stmeasurement
• Family history of congenital renal disease ▪ Average of 2nd and 3rd measurement used to determine the child’s
• Solid-organ transplant BP category
• Malignancy or BM transplant ▪ BP is checked 3 times, averaging the results
▪ Confirmed on repeat visits
• Treatment with drugs known to raise BP
• Other systemic illnesses known to be assocwith HTN AMBULATORY BLOOD PRESSURE MONITORING
[neurofibromatosis, TS etc] ▪ The patient wears a device that records BP every 20-30 min,
• Evidence of elevated intracranial pressure throughout a 24 hr period, during usual daily activities, including
sleep. This monitoring allows calculation of the mean daytime BP,
sleep BP, and mean BP over 24 hr.
▪ Strongly correlated with target organ damage CENTRAL
▪ Indications ▪ Intracranial mass
• Diagnose white coat hypertension (elevated office BP but normal ▪ Hemorrhage
ambulatory BP) as well as masked hypertension (normal office ▪ Residual following brain injury
BP but elevated ambulatory BP). ▪ Quadriplegia (dysautonomia)
▪ Sleep disordered breathing
• Evaluating effectiveness of antihypertensive therapy ENDOCRINE
• High-risk patient populations, such as children with CKD, solid- ▪ Hyperthyroidism
organ transplant, diabetes mellitus, and severe obesity ▪ Congenital adrenal hyperplasia (11β-hydroxylase and 17-hydroxylase
defect)
▪ Limitations
▪ Cushing syndrome
• Not tolerated by younger children and some children with ▪ Primary hyperaldosteronism
developmental delay ▪ Apparent mineralocorticoid excess
▪ Glucocorticoid remedial aldosteronism (familial aldosteronism type 1)
▪ Glucocorticoid resistance (Chrousos syndrome)
▪ Pseudohypoaldosteronism type 2 (Gordon syndrome)
▪ Pheochromocytoma Other neural crest tumors (neuroblastoma,
ganglioneuroblastoma, ganglioneuroma)
▪ Liddle syndrome
▪ Geller syndrome

CONDITIONS ASSOCIATED WITH TRANSIENT OR


INTERMITTENT HYPERTENSION IN CHILDREN
RENAL
▪ Acute postinfectious glomerulonephritis
▪ Henoch-Schönlein purpura with nephritis
OFFICIAL ABPM READING ▪ Hemolytic-uremic syndrome
▪ Acute kidney injury
▪ After renal transplantation (immediately and during episodes of rejection)
▪ Hypervolemia
▪ Pyelonephritis
▪ Renal trauma
▪ Leukemic infiltration of the kidney
DRUGS AND POISONS
▪ Cocaine
▪ Oral contraceptives
▪ Sympathomimetic agents
▪ Amphetamines
▪ Phencyclidine
▪ Impression: Nocturnal and daytime systolic and diastolic ▪ Corticosteroids and adrenocorticotropic hormone
hypertension with dipping nocturnal systolic and diastolic blood ▪ Cyclosporine, sirolimus, or tacrolimus treatment after transplantation
pressure pattern. ▪ Licorice (glycyrrhizic acid)
▪ Lead, mercury, cadmium, thallium
▪ Dipping: ( (mean wake-mean sleep)/mean wake BP ) x 100% ▪ Antihypertensive withdrawal (clonidine, methyldopa, propranolol)
Systolic Dipping: 23.5% ▪ Vitamin D intoxication
Diastolic Dipping: 25.6% CENTRAL
PHYSIOLOGY ▪ Increased intracranial pressure
▪ Guillain-Barré syndrome
Blood Pressure – the force exerted by the blood against any unit ▪ Burns
area of vessel wall ▪ Familial dysautonomia
▪ Stevens-Johnson syndrome
▪ Posterior fossa lesions
▪ Porphyria
▪ Poliomyelitis
▪ Encephalitis
▪ Spinal cord injury (autonomic storm)
MISCELLANEOUS
▪ Preeclampsia
CONDITIONS ASSOCIATED WITH CHRONIC HYPERTENSION ▪ Pain, anxiety
RENAL
▪ Hypercalcemia
▪ After coarctation repair
▪ Recurrent pyelonephritis/renal scarring ▪ White blood cell transfusion
▪ Chronic glomerulonephritis ▪ Extracorporeal membrane oxygenation
▪ Prematurity
▪ Congenital dysplastic kidney Renal disease (e.g., chronic glomerulonephritis, reflux or
▪ Polycystic kidney disease
obstructive nephropathy, hemolytic-uremic syndrome, polycystic
▪ Vesicoureteral reflux nephropathy
▪ Segmental hypoplasia (Ask-Upmark kidney) Obstructive kidney kidney disease, congenital anomalies of the kidney and urinary
disease tract) and renovascular hypertension account for approximately
▪ Renal tumors 90% of children with secondary hypertension.
▪ Renal trauma Renal parenchymal disease and renal artery stenosis lead to
▪ Systemic lupus erythematosus (other connective tissue diseases) water and sodium retention thought to be, in part, secondary to
VASCULAR increased renin secretion.
▪ Coarctation of thoracic or abdominal aorta Several endocrinopathies are associated with hypertension,
▪ Renal artery lesions (stenosis, fibromuscular dysplasia, usually those involving the thyroid, parathyroid, and adrenal
thrombosis, aneurysm) glands.
▪ Umbilical artery catheterization with thrombus formation Systolic hypertension and tachycardia are common in
▪ Neurofibromatosis (intrinsic or extrinsic narrowing for vascular hyperthyroidism; DBP is not usually elevated.
lumen) Hypercalcemia , whether secondary to hyperparathyroidism or
▪ Renal vein thrombosis
other causes, often results in mild elevation in BP because of an
▪ Vasculitis (ANCA associated, polyarteritis nodosa, Takayasu
increase in vascular tone.
arteritis)
▪ Arteriovenous shunt Adrenocortical disorders (e.g., aldosterone-secreting tumors,
▪ Williams-Beuren syndrome sodium-retaining congenital adrenal hyperplasia,
▪ Moyamoya disease Cushing syndrome) may produce hypertension in patients with
increased mineralocorticoid secretion.

2 PEDIATRICS: HYPERTENSION, UTI, WILMS TUMOR AISH


Pheochromocytomas are catecholamine-secreting tumors that ACCORDING TO AGE GROUP
give rise to hypertension because of the cardiac and peripheral
vascular effects of epinephrine and norepinephrine.
Rarely, secondary hypertension can be caused by
pseudohyperaldosteronism , which leads to elevated BP in the
face of a suppressed renin level.
Altered sympathetic tone can be responsible for acute or
intermittent elevation of BP in children with Guillain-Barré
syndrome, poliomyelitis, burns, and StevensJohnson syndrome.
Intracranial lesions also affect sympathetic outflow from the
central nervous system.
A number of drugs of abuse , therapeutic agents , and toxins may
cause hypertension.
→Cocaine may provoke a rapid increase in BP and can result in
seizures or intracranial hemorrhage.
→Phencyclidine causes transient hypertension that may become
persistent in chronic abusers
→Tobacco use may also increase BP.
→Sympathomimetic agents used as nasal decongestants,
appetite suppressants, and stimulants for attention-deficit
disorder produce peripheral vasoconstriction and varying degrees
of cardiac stimulation.
→Oral contraceptives should be suspected as a contributor to
elevated BP in adolescent girls, although the incidence is lower
with the use of low-estrogen preparations.
→Immunosuppressant agents such as cyclosporine and
tacrolimus cause hypertension in organ transplant recipients, and
the effect is exacerbated by the co-administration of
corticosteroids.
→BP may be elevated in patients with poisoning by a heavy metal
(lead, cadmium, mercury).
Primary Hypertension - These patients often are overweight,
have a strong family history of hypertension, and have BP values
at, or only slightly above, the 95th percentile for age.
Isolated systolic hypertension is also more consistent with primary
hypertension, whereas diastolic hypertension may suggest a
secondary cause.
Elevated uric acid levels may play a role in the pathophysiology
of primary hypertension, and proof-of-concept studies have
confirmed that lowering of uric acid levels results in lower BP in
overweight youth with hypertension or prehypertension.

CLINICAL FINDINGS IN PATIENTS WITH EXCESS ALDOSTERONE

RENAL DISEASE AND RENOVASCULAR HYPERTENSION

▪ Renal disease (e.g., chronic glomerulonephritis, reflux or


obstructive nephropathy, hemolytic-uremic syndrome, polycystic
kidney disease, congenital anomalies of the kidney and urinary
tract) and renovascular hypertension account for approximately
90% of children with secondary hypertension.

CAUSES OF CHILDHOOD HYPERTENSION

3 PEDIATRICS: HYPERTENSION, UTI, WILMS TUMOR AISH


CLINICAL MANIFESTATIONS
Children and adolescents with primary hypertension are usually
asymptomatic; the BP elevation is usually mild and is detected during a
routine examination or evaluation before athletic participation. → these
children may also be obese
Children and adolescents with acute severe hypertension , in contrast,
present with BP elevation well above stage 2 hypertension and severe
symptoms that may represent acute target-organ injury.
Subclinical hypertensive target-organ injury is a common clinical
manifestation in children with primary hypertension.

PHYSICAL EXAM
FINDINGS TO LOOK FOR ON PHYSICAL EXAMINATION IN PATIENTS
WITH HYPERTENSION

4 PEDIATRICS: HYPERTENSION, UTI, WILMS TUMOR AISH


The 1st step in diagnosing hypertension is recognition of elevated Doppler ultrasonography is not recommended when screening for
BP. renovascular hypertension in the 2017 AAP guideline except in
BP readings taken in the office should be compared to normative selected patients
BP tables, indexed by height and sex, to ensure that the patient CAUSES OF RENOVASCULAR HTN
is normotensive.
Elevated office BP readings should be confirmed using ABPM to Request for lipid panel and
identify children with white coat hypertension, who may not glucose
require further evaluation a sleep history should be
In all patients, a careful history and physical examination are obtained in children with
warranted. confirmed hypertension to
Birth history should be documented to screen for prematurity and screen for sleep-disordered
other perinatal events that may affect later BP. breathing , an entity that is
A family history for metabolic disease, renal disease, early associated with high BP,
cardiovascular events, and other forms of secondary particularly in overweight
hypertension should be obtained. children.
Growth parameters should be determined to detect evidence of Left ventricular hypertrophy
chronic disease. BP should be obtained in all 4 extremities to (LVH) is the most common
detect coarctation (thoracic or abdominal) of the aorta. manifestation of target organ
Unless the history and physical examination suggest another damage in hypertensive disorder
cause, children with confirmed hypertension should have an
evaluation to detect renal disease, including urinalysis,
electrolytes, blood urea nitrogen, creatinine, and complete blood
count. TREATMENT
Standard renal ultrasound should be considered in patients with
▪ The mainstay of therapy for children with asymptomatic mild
a higher suspicion of secondary hypertension to assess for
hypertension without evidence of target-organ damage is
discrepancies in renal size, structural abnormalities, and other
potential causes of hypertension. therapeutic lifestyle modification with dietary changes and regular
Measuring serum potassium is essential because exercise.
o hypokalemia may be present in renovascular hypertension ▪ Weight loss is the primary therapy in obesity-related hypertension.
and many monogenic forms of hypertension (including It is recommended that all hypertensive children have a diet
Liddle syndrome, glucocorticoid remedial aldosteronism, increased in fresh fruits, fresh vegetables, fiber, and nonfat dairy
and apparent mineralocorticoid excess) and reduced in sodium.
o hyperkalemia may be seen in Gordon syndrome. DASH - DIETARY APPROACHES TO STOP HYPERTENSION
▪ Demonstrated in adults: BP lowering benefits of a diet that was high
CLINICAL EVALUATION OF CONFIRMED HYPERTENSION in fresh fruits, vegetables, whole grains and low-fat dairy products.
▪ Pilot study in adolescents: The efficacy of a clinic based behavioral
nutrition intervention emphasizing a DASH type diet for adolescents
with elevated blood pressure.
▪ [+] Salt intake control.

Renovascular hypertension is often associated with other


diseases but may be isolated INDICATIONS FOR ANTIHYPERTENSIVE DRUG THERAPY IN
Magnetic resonance (MR) or computed tomography (CT)
CHILDREN
angiography can reveal renal artery stenosis, but formal
intraarterial angiography may be needed to detect intrarenal ▪ Symptomatic hypertension
vascular stenoses and in infants and young children, in whom ▪ Secondary Hypertension
noninvasive imaging techniques often are not helpful because of ▪ Hypertensive target-organ damage
small vessel size. ▪ Diabetes [types 1 &2]
▪ Persistent hypertension despite non-pharmacologic measures

5 PEDIATRICS: HYPERTENSION, UTI, WILMS TUMOR AISH


The dose can then be increased until the goal BP is achieved.
Once the highest recommended dose is reached, or if the child
develops side effects, a 2nd drug from a different class can be
added.
angiotensin-converting enzyme inhibitors (ACEIs), angiotensin
receptor blockers (ARBs), thiazide diuretics, and calcium channel
blockers are generally considered acceptable initial agents for
use in children.
The choice of antihypertensive agent for a patient should be
tailored to the etiology of that patient's hypertension whenever
possible.

TARGET-ORGAN ABNORMALITIES IN CHILDHOOD HYPERTENSION


▪ Target-organ abnormalities are commonly associated with
hypertension in children and adolescents.
▪ LVH is the most prominent evidence of target-organ damage.
▪ Pediatric patients with established hypertension should have ECG
assessment of LV mass at diagnosis and periodically thereafter.
▪ The presence of LVH is an indication to initiate or intensify
antihypertensive therapy.

PHARMACOLOGIC THERAPY OF CHILDHOOD HYPERTENSION


▪ The goal for antihypertensive treatment in children should be
reduction of BP to < 95th percentile unless concurrent conditions
are present, in which BP should be lowered to < 90 percentile
▪ Severe, symptomatic hypertension should be Treated with
intravenous antihypertensive Drugs.

^^if you can’t read this check p.9829 of Nelson 21st ed


BOTTOMLINE: DETERMINE UNDERLYING DISEASE!! 😊
URINARY TRACT INFECTION
▪ Infection = bacterial growth

SYMPTOMS
Major factors that are in BP regulation…
▪ Fluid volume
▪ Vascular resistance
▪ Cardiac output

DEFINITIONS

KIDNEY DAMAGE
▪ Focal or generalized persistent kidney damage
▪ On DMSA scan, persistent kidney damage is evidenced by one or
more photon-deficient areas and/or generalized reduction in DMSA
CAUSES Vascular Other causes uptake several months after the diagnosis of a UTI.
Fibromuscular Takayasu Arteritis Radiation
Dysplasia
PREVALENCE
Syndromic Causes Polyarteritis nodosa UAC
NFT1 Kawasaki Disease Trauma
Under age 1 yr
Tuberous Sclerosis Other Vasculitis Congenital Rubella ▪ Males
Williams Syndrome Extrinsic Transplant Renal ▪ febrile infants is 7%.
Compression Artery Stenosis ▪ Uncircumcised males
Marfan Syndrome Neuroblastoma
Other Syndromes Wilms Tumor

6 PEDIATRICS: HYPERTENSION, UTI, WILMS TUMOR AISH


Beyond 1-2 yr Uropathogenic E. coli
▪ Female preponderance ▪ Presence of bacterial pili or fimbriae on the bacterial surface
o By the age of 5 yr ▪ Type II fimbriae (mannose-resistant)
o Infancy ▪ Receptor (Gal1-4Gal oligosaccharide fraction) for Type II fimbriae
o Toilet training present in uroepithelial cell membrane and RBC
o Onset of sexual activity ▪ Type II fimbriae – P fimbriae
▪ Male: female ratio 1:10 o Can agglutinate P blood group erythrocytes
▪ afebrile symptomatic UTIs in children over age 1 yr is ~8% o Cause pyelonephritis
▪ Compete with nutrients
ETIOLOGY
▪ Caused primarily by colonic bacteria
▪ Escherichia coli causes 54–67% of all UTIs, followed by Klebsiella
spp. and Proteus spp., Enterococcus, and Pseudomonas
▪ Other bacteria known to cause UTIs include Staphylococcus
saprophyticus, group B streptococcus, and, less commonly,
Staphylococcus aureus, Candida spp., and Salmonella spp

DIAGNOSIS
▪ Fever is the most common symptom among infants and young
children
▪ UTI is <5% of febrile infections in children
▪ Most useful indication in children below 24 mos
o Fever above 40C
o Previous history of UTI
o Fever for more than 24h
o Suprapubic tenderness
o Ill appearance
o No other source of fever
o Lack of circumcision
▪ Combined predictors more useful than individual findings
o Fever exceeding 39 C for more than 48 h
o Without another source of fever
PYELONEPHRITIS
▪ Fever may be the only manifestation; particular consideration
should occur for a temperature > 39°C without another source
lasting more than 24 hr for males and more than 48 hr for females.
▪ Pyelonephritis is the most common serious bacterial infection in
infants younger than 24 mo of age who have fever without an
obvious focus.

URINE SAMPLE COLLECTION


▪ Urine culture remains the reference standard for diagnosis UTI

RAPID PATIENT TESTS


CYSTITIS ▪ Dipstick testing of urine for leukocyte esterase and nitrite or urinary
▪ Often caused by E. coli; it also has been attributed to adenovirus microscopy for pyuria or bacteria CANNOT replace culture for
types 11 and 21. diagnosis
▪ Adenovirus cystitis is more common in boys; it is self-limiting, with
hematuria lasting approximately 4 days.
▪ Patients receiving immunosuppressive therapy (e.g., solid-organ or
bone marrow transplantation) are at higher risk for hemorrhagic
cystitis; adenoviruses and polyomaviruses (i.e., JC virus and BK
virus)

PATHOPHYSIOLOGY
▪ Ascending Infections
o fecal flora, colonize the perineum, and enter the bladder via the
urethra.
o In uncircumcised males, pathogens arise from the flora beneath
the prepuce
o In some cases, the bacteria causing cystitis

7 PEDIATRICS: HYPERTENSION, UTI, WILMS TUMOR AISH


ADDITIONAL TESTS
BLOOD CULTURE
▪ Bacteremia in the setting of pyelonephritis is reported to occur in 3–
20% of patients
▪ Most common in infants less than 90 days old (with rates decreasing
with increasing age in the first 90 days) and in any child with
obstructive uropathy
▪ If the patient appears to be ill at presentation

TREATMENT

WILMS TUMOR
▪ also known as nephroblastoma
▪ most common primary malignant renal tumor of childhood
o 6% of pediatric malignancies
o >95% of kidney tumors in children
o 75% of the cases occur
o In children <5 yr old, with a peak incidence at 2-3yr
o Incidence of bilateral Wilms Tumor is 7%
▪ Second most common malignant abdominal tumor in childhood
after neuroblastoma
▪ In 8–10% of patients, Wilms tumor is observed in the context of
hemihypertrophy, aniridia, genitourinary anomalies, and a variety of
rare syndromes, including Beckwith-Weidemann syndrome (BWS)
and Denys Drash syndrome
▪ The most common sites of metastases
o lungs
o regional lymph nodes
o liver

HISTOLOGY
▪ The classic Wilms tumor is made up of varying proportions of
blastemal, stromal, and epithelial cells, recapitulating stages of
normal renal development.

8 PEDIATRICS: HYPERTENSION, UTI, WILMS TUMOR AISH


▪ derived from incompletely differentiated renal mesenchyme, and ▪ Rapid abdominal enlargement and anemia occur because of
tumors are typically composed of cells reminiscent of the bleeding into the renal parenchyma or pelvis.
undifferentiated and partially differentiated cells that normally arise ▪ Wilms tumor thrombus extends into the inferior vena cava (IVC) in
from renal mesenchyme 4– 10% of patients and rarely into the right atrium; dislodgment of
the intravascular tumor may produce a fatal pulmonary embolism.

DIAGNOSIS
▪ An abdominal mass in a child should be considered malignant until
diagnostic imaging, laboratory findings, and pathology can define its
true nature

IMAGING STUDIES

▪ Distinguish renal mass from adrenal masses (e.g. neuroblastoma)


and other masses in the abdomen
▪ Wilms tumor: focal areas of necrosis or hemorrhage and
hydronephrosis because of obstruction of the renal pelvis by the
tumor
▪ Bone scan is performed if the histologic diagnosis confirms clear
cell sarcoma of the kidney or rhabdoid tumor of the kidney, to look
for bone metastasis
▪ Positron emission tomography (PET)/CT scanning: regional spread
and metastatic lesions can be visualized

HISTOPATHOLOGY
▪ Diagnosis is usually made by imaging studies and confirmed by
histology at the time of nephrectomy.
▪ Although biopsy is a reliable diagnostic tool, it is discouraged since
▪ Foci of benign, undifferentiated mesenchyme (nephrogenic rests) it results in disease upstaging
that persist abnormally in the kidney into postnatal life are observed ▪ Core needle biopsy
in approximately 1% of children in the general population, but are →posterior approach (to limit contamination of the peritoneal cavity)
present in up to 90% of children who have a family history of Wilms should be performed in cases of unusual presentation (>10 yr old,
tumor, develop bilateral tumors, or display features of Wilms tumor– signs of infection, inflammation) or unusual imaging findings
related syndromes (significant adenopathy, no renal parenchyma seen, intratumoral
▪ Nephrogenic rests usually regress or differentiate, but those that calcification)
persist can become malignant

NEPHROGENIC RESTS
▪ Abnormal persistence of embryonic cells capable of developing into
Wilms tumor
▪ Identify patients at risk of contralateral Wilms tumor
o Perilobar rests: circumscribed rests located at periphery of a renal
lobe
o Intralobar rests: located in center of arenal lobe

CLINICAL PRESENTATION RISK STRATIFICATION


▪ The most common initial clinical presentation is the incidental ▪ The most powerful prognostic factor for outcome in WT is the
discovery of an asymptomatic abdominal mass by parents while histology of the tumor.
bathing or clothing an affected child or by a physician during a ▪ NWTS and COG define WT as favorable histology (FH) if anaplasia
routine physical examination is not identified
▪ At presentation, ▪ Increasing age has been shown to be correlated with decreased
o Large mass prognosis by both cooperative groups
o Hypertension in about 25% ▪ Nephrectomy alone is sufficient for patients
o Abdominal pain (40%), gross painless hematuria (18%), and
constitutional symptoms such as fever, anorexia, and weight loss
are other findings at diagnosis
9 PEDIATRICS: HYPERTENSION, UTI, WILMS TUMOR AISH
▪ Patients with diffuse anaplasia have a particularly poor outcome
and are treated with intensive chemotherapy regimens that include
vincristine, cyclophosphamide, doxorubicin, etoposide, carboplatin,
and ifosfamide, in addition to radiation therapy

RADIATION THERAPY
▪ Regional lymph node metastases, residual disease after surgery,
or tumor rupture receive radiation therapy to the flank or abdomen,
and those with lung metastases receive radiation therapy to the
lungs.
▪ Rapid response of lung metastases to chemotherapy may
eliminate the need for lung radiation

PROGNOSIS
▪ Approximately 15% of favorable-histology and 50% of
anaplastichistology Wilms tumors relapse; most relapses occur
early (within 2 yr of diagnosis)
▪ Overall survival of children with Wilms tumor exceeds 90%, with
some prognostic factors (low stage, favorable histology, young age,
low tumor weight) conferring even better outcomes.

COMPLICATIONS
▪ Late complications are a consequence of treatment type and
intensity; the use of radiotherapy and anthracyclines increase the
risk of these complications

Hello Malabo talaga to sa ppt. pls check it na lang din sa ppt. below is
a table from nelson

TREATMENT
▪ Surgery and chemotherapy with or without radiotherapy
▪ Use of multimodality treatment and multiinstitutional cooperative
group trials has dramatically improved the cure rate of Wilms tumor
from 90%
▪ Early surgery provides accurate diagnosis and staging and can
facilitate risk-adapted therapy
▪ Preoperative chemotherapy can make surgery easier and reduces
the risk of intraoperative tumor rupture and hemorrhage.

SURGERY
▪ Radical nephrectomy , with meticulous dissection to avoid rupture
of the tumor capsule, and lymph node sampling despite the absence
of abnormal nodes on preoperative imaging studies or
intraoperative assessment.
▪ is performed in patients with bilateral disease or those with unilateral
Wilms tumor and predisposing syndrome such as the Denys-Drash
and WAGR syndromes, to minimize the risk of future renal failure

CHEMOTHERAPY
▪ Stage I and II disease receive chemotherapy with 2 drugs, If may di kayo basa na table just check the ppt or Nelson
vincristine and actinomycin D (also called dactinomycin), every 1-3 21st ed huhu sorry I’m tired na.
wk for a total of 18 wk (regimen EE4A ). HTN p.9803
▪ Stage III or IV disease receive chemotherapy with 3 drugs UTI p.10943
(vincristine, doxorubicin, and actinomycin D) every 1-3 wk for a total Wilms Tumor p.10524
of 24 wk (regimen DD4A ) and radiation therapy.

10 PEDIATRICS: HYPERTENSION, UTI, WILMS TUMOR AISH


PEDIA 3A – NEPHRO GN - DR. MATHEUS (SAMPLEX)
(Sabayan nyo ng trans to. Gamit ko yung 3B trans. Pero paulit ulit lang naman yung mga
tanong. At madaming cases.Napaghalo ko yung majors and shiftings. Pero umuulit lang  HSP
talaga yung mga tanong. STUDY AT YOUR OWN RISK.) - IgA deposits in mesangial cells of glomerulus WITH systemic
manifestations
Mga clue lang from samplex dahil nalilito din ako dito:  HUS
- Microangiopathic haemolytic anemia
 UTI - Hemoglobinuria
- imaging: ultrasound of kidneys and urinary bladder - tea colored urine (yung PIGN and IgA nephro may tea colored
- confirm: urine culture and sensitivity din pero meron silang edema and URTI respectively)
- better sensitivity: pyuria and nitrite test and wbc esterase test  Hemorrhagic cystitis
- to diagnose: do a urine culture - bright red urine with blood clots
- neonates: sepsis syndrome MAJOR EXAMS AND SHIFTINGS
- infant: fever
- lower UTI: Amoxicillin
 Nephrotic syndrome
HEMATURIA IS GLOMERULAR
- hematuria  Urinalysis is positive for significant hematuria and proteinuria
- massive protein loss in the urine
- initial diagnostic study: urinalysis MICROSCOPIC HEMATURIA
- DOC: prednisone  Isolated microscopic hematuria: Long term follow up is necessary to
- pag may sinabing varicella sa case: give varicella immune globulin rule out progressive renal disease
- pag vaccine tas naka steroid si patient: conjugate pneumococcal  Isolated microscopic hematuria: correlates with best renal prognosis
- to determine presence of nephrotic range proteinuria: urine  Case: 10y/o female. Microscopic hematuria from 10-20 rbc/hpf on 3
protein/creatinine ratio diff occasions. No complaints by the patient. PE findings were
- manifestation is caused by: hyperalbuminemia
normal. Next step: repeat urinalysis and observe
 PIGN
 Alport's syndrome: initially presents with microscopic hematuria
- edema, swelling
- confirm: urinalysis, complement 3, ASO titer, creatinine without systemic manifestations in vhildren but may persist in
- strengthen diagnosis: latent period adulthood with associated sensorineural loss
 Glomerulonephritis  CASE: 11 y/o male consulted due to isolated microscopic hematuria
- Cardinal manifestation: hematuria ranging from 10-20 RBC/Hpf on 3 separate occasions. There are no
- Lab exam to diagnose: Urinalysis complaints by the patient and PE findings were essentially normal.
- GN is considered when: urinalysis has significant hematuria and As part of hx taking it is important to ask for: Family hx of chronic
proteinuria kidney dse
- Complication: hypertension  CASE: On a routine examination done for school entrants, a 6 y/o
 IgA nephropathy male was found to have microscopic hematuria ranging from 10-
- IgA deposition WITHOUT systemic disease
20rbc/hpf and proteinuria of +2. The patient was asymptomatic.
- assoc with URTI
What would you ask if you are thinking of glomerulonephritis?:
history of previous infection

1 ejg
UTI  Proteinuria: greater than 40 mg/m2/hr
 In children: >40,000 CFU from a suprapubic sample  Minimum: 40 mg/m2/hr
 In children: Initial renal imaging: ultrasound of the kidneys and  Minimum duration of tx with high dose steroid prior tapering: 4
urinary bladder weeks
 Case: 2y/o girl, had 2 eps of UTI in the past 4mos. Urine culture was  Patients diagnosed with nephrotic syndrome should be started with
E. coli >100,000 CFU/ml. Renal imaging study: ultrasound of the prednisone: 60 mg/m2/day (sagot sa iba ay 20 mg/m2/day, di ko
kidneys and urinary bladder alam kung alin ang tama. Pero feeling ko yung 60 yung tama.
 Neonates: Sepsis is the most common manifestation of UTI; sepsis Hahaha. Walang sisihan ah)
syndrome  Adult nephrotic syndrome: Membranous GN
 Highest possibility that UTI is present: positive test for WBC esterase  Case: 8y/o girl, new onset swelling around the eyes. Periorbital,
and nitrite test sacral, pretibial edema. BP is 96/64. Most appropriate initial dx
 Better sensitivity for diagnosing true UTI: pyuria, +nitrite test, +WBC study: urinalysis
esterase test, bacteriuria  Case: 2y/o male, pale looking, generalized edema. Urinalysis
 Case: 3y/o, abd pain, fever, pain on urination, lower abdominal showed Albumin of +4. Serum albumin was 2.2 gm/dL. Cholesterol
tenderness, right sided costo-vertebral angle tenderness. To was 2x above normal. 24 hr urine protein was 44 mg/m2/hr. ASO
confirm diagnosis: urine culture and sensitivity titer was <200 IU. Start the patient on: Prednisone (DOC)
 Case: 3 month old boy, high grade fever for one day. Urinalysis was  Case: 6y/o male, severe abd pain. 10 days PTC, sudden onset of
done and revealed Dark colored urine with pus cell of 50-60/hpf, facial edema. Edema persisted, gradually involved LE with distention
nitrite test+3 and WBC esterase test +2. Next step: do a urine of abdomen. UO decreased, no change in urine color. Lab exam:
culture (used to diagnose) Urinalysis
 In infant: seen as fever  (Above case) cause of severe abd pain: primary peritoneal infection
 Suprapubic aspiration: any number of colonies  (Above case) strenghten diagnosis: presence of massive proteinuria
 Catheterized urine: >/= 10,000 colonies  Case: 5y/o girl with generalized swelling (edema). Urine protein
 Clean catch urine: >/= 100,000 single colony 400mg/dL (heavy proteinuria, coz >200), Albumin 1.6 g/dL
 Urine bag urine specimen: > 100,000 2 or more organisms (hypoalbuminemia, coz <2.5g/dL), Cholesterol 360 mg/dL
 Lower UTI: Amoxicillin (hypercholesterolemia). Dx: Nephrotic Syndrome
 For a case of UTI, in what situation would you advice advanced renal  Case: 3y/o male, in relapse, on steroid therapy exposed to a cousin
imaging?: Patients with midline anomalies at the lumar area with variclella 2 days PTC. Next step: give varicella immune globulin
 Least reliable indicator for the presence of UTI: pus cells immediately
 Usual organism isolated for UTI in children: E. coli  Immunocompromised because: serum albumin level is low;
immunoglobulins are lost in urine
NEPHROTIC SYNDROME  Part of management: provide adequate amount of protein intake
 Main event that leads to cascade of manifestations: massive protein for growth
loss in urine  Diagnosed Nephrotic syndrome, initial duration of prednisone tx
 Minimal Change Nephrotic Syndrome (MCNS): male, 2-6 years old, based on APN: 12 weeks
most steroid-responsive

2 ejg
 True of the APN protocol: associated with a longer remission period  Case: 4y/o boy sustained shallow wound from protruding nail 16
and fewer relapses days PTA. 12 days PTC, wound got infected. 4 days PTC, puffy
 Diagnosed Nephrotic syndrome, initial duartion of prednisone tx eyelids with gross hematuria and pedal edema. Latent period for
based on ISKDC: 8 weeks this case: 8 days
 ISKDC criteria: heavy proteinemia, edema, hyperlipidemia. Except:  Is an immune complex dse. Injury to kidney is due to: deposition of
hyperalbuminuria circulating immune complex in the kidneys
 Vaccine given in nephrotic patients on steroid therapy: conjugate  TRUE: antibiotic tx is mandatory regardless of the etiology, most
pneumococcal viral causes have a shorter latent period, prognosis of acute post
 CASE: 7 month old male infant is suspected to have nephrotic strep GN is very good (NOT TRUE: a complicated course is
syndrome, since timed urine collection is difficult to do. The next anticipated for post-viral GN)
best thing to determine presence of nephrotic range proteinuria is  APGN: associated with hypertension; trace back 10 days for history
to request for a: urine protein/creatinine ratio of previous infection; formation of immune complex (antigen-
 The clinical manifestation of nephrotic syndrome is primarily caused antibody reaction)
by: hyperalbuminemia  PIGN: prognosis of APGN is very good; high blood pressure
 AGN with NORMAL creatinine: hypertension, pulmonary
POST STREP GN/POST INFECTIOUS GN congestion, edema
 Average latent period: 10-14 days  AGN with ABNORMAL creatinine: hematuria
 Case: 6y/o, sustained wound 16 days PTA. 12 days PTA, wound got  To strengthen the diagnosis of post-GN, establish presence of:
infected, no meds taken. 2 days PTC, had puffy eyelids, pedal latent period
edema, gross hematuria. Latent period is: 10 days  Complications: hypertensive crisis, volume overload, renal failure.
 Case: 6y/o male, facial edema. 5 days PTC, there's sudden onset of Except: edema
facial edema, distention of abdomen, edema of LE + tea colored  CASE: A 3 y/o boy has been coming back regularly for the past 6
urine, oliguria, headache, vomiting. On PE, facial edema, positive months due to microscopic examination, repeated urinalysis done in
fluid wave, decreased breath sounds on both lung fields, grade 2 the past revealed only significant hematuria. There is no family
pitting edema. Lab exams to confirm dx: urinalysis, complement 3, history of renal dse. PE is normal what is your course of action in the
ASO titer mgnt of the case?: continue to monitor the patient and not any
 Case: 7y/o female, (same as above patient, pinalitan lang yung age signs of renal progression before doing more intense work up
and sex): urinalysis, CBC, creatinine, complement 3, ASO titer  (Clue: Actually di ko alam kung pano nakukuha yung latent period.
 Case: patient had sore throat 15 days ago, not treated. 4 days PTA, Pero ang napansin ko lang, ima-minus mo lang yung last 2 numbers
puffiness of eyelids. Progressed to facial and pedal edema. Latent na ‘PTC or PTA’ okaya kung dalawa lang yung given na ‘PTC or PTA’
period: 11 days sa case, saka mo makukuha yung latent period. Haha. Di na ko nag-
 Case: 5y/o male, 12 days PTA, empty milk can was thrown at him abalang maghanap ng ratio at di ko pa din to naaaral. O baka ako
on his right leg resulting to a shallow abrasion. 9 days PTA, abrasion lang may hindi alam kung pano yun nalalaman?! Hahaha. Sarreh.
have erythematous borders with some purulent discharge. 2 days Basahing mabuti yung tanong. Minsan in word nakalagay yung
PTA, puffiness of eyelids then few hours developed generalized numbers. #testtaking)
seizures. Latent period for this case: 7 days

3 ejg
GLOMERULONEPHRITIS  IgA deposits in mesangial cells of glomerulus WITH systemic
 Cardinal manifestation: hematuria manifestations
 Lab exam to diagnose: Urinalysis (5th case in post infectious GN ^)
 Complication that could have caused seizure: hypertension (5th WILM'S TUMOR
case in post infectious GN ^)  Case: 7y/o male, abdominal mass. 2mos PTA, had hard mass on
 GN is considered when: urinalysis has significant hematuria and superior part of the left kidney. 3 days PTA, complains of vague
proteinuria abdominal pain. On CT scan, abdomen showed large left renal mass
 Complication: hypertension probably Wilm's tumor with lymph nodes on periaortic area.
Classification of the patient: Stage 3
ASO TITER  Case: wilm's tumor patient had nephrectomy. All tumor infiltrates at
 Strep infection, increase after: 2 weeks operative area not removed. No metastasis in other organ system:
 Sensitivity is lower for strep pyodermas Stage 3
 (Above patient) tumor weighed 650g. There's hyperchromatism and
PROTEINURIA anaplasia of cells. Has poor prognosis based on: stage of patient,
 Significant protein in the urine: 24 hour urine protein/albumin size and wt of tumor, unfavorable histo type(AOTA)
determination  Renal malformation associated: Duplicating renal collecting system
 Case: previously healthy 8y/o boy, abrupt onset of facial swelling,  Seen in the bone: Stage 4
edema, periorbital and facial swelling, urine is normal except for  Case: Condition did not improve despite intensive measures. Urine
presence of protein. Normal BUN and creatinine. Next step in is cola colored with bulging on left side
management: give oral prednisone (DOC: prednisone or  Bilateral: Stage 5
prednisolone)  Not treated with radiation: Stage 1
 Associated with wilm’s tumor except: reflux (kasama yung:
IGA NEPHROPATHY hypoplastic kidney, hydronephrosis, ectopic kidney)
 Case: 9y/o male, tea colored urine, cough, colds, low grade fever,
urinalysis showed too many to count rbc with low grade proteinuria, RISK FACTOR IN THE DEVELOPMENT OF RENAL VENOUS THROMBOSIS
happened before patient had acute upper respiratory tract infection  Dehydration
 Recurrence of gross hematuria whenever patient suffers an URTI  Respiratory distress syndrome (RDS)
 IgA deposition WITHOUT systemic disease  Renal venous thrombosis: pathophy involves microangiopathic
 Poor renal prognosis: progressive and heavy proteinuria, persistent hemolytic anemia
hypertension, diminished renal function
 Indicator of prognosis: development of proteinuria HEMOLYTIC UREMIC SYNDROME (HUS)
 Anemia, gross hematuria, renal failure, history of diarrhea
HENOCH SCHONLEIN PURPURA NEPHRITIS/HSP  Microangiopathic haemolytic anemia
 abdominal pain, arthritis, purpura rash on LE and GN are suggestive  Case: 1y/o male, tea colored urine 3 days duration. Hx of diarrhea,
of: HSP sudden onset of pallor, decreased UO for more than 24hrs.
Urinalysis showed tea colored +4 for blood protein is trace and rbc

4 ejg
of 5-6/hpf. Bun and creatinine were 4x above normal. Impression: 24 HOUR URINE CREATININE CLEARANCE
HUS  Most sensitive tool to assess renal function in a stable state
 Hemoglobinuria
24 HOUR URINE ALBUMIN DETERMINATION
PENICILLIN  One way to distinguish tubular vs. glomerular proteinuria
 Significant hematuria and proteinuria with low C3 and +ASO titer.
Start penicillin. TRUE OR FALSE
 For primary peritonitis secondary to nephrotic syndrome 1. Postinfectious glomerulonephritis is antibody dependent
cytotoxicity. FALSE (immune mediated)
PRIMARY PERITONITIS 2. Microscopic hematuria is not indicative of postinfectious GN. FALSE
 Severe abdominal pain, periumbilical in location with some (gross or microscopic hematuria)
abdominal guarding associated with fever 3. Hematuria is the hallmark of nephrotic syndrome. FALSE (edema)
4. Steroid treatment for nephrotic syndrome is 10-week regimen.
CYSTITIS, HEMORRHAGIC FALSE (12 weeks)
 Case: 7y/o male, dysuria. Recent cough, colds, no fever. Gross 5. Steroid sensitive NS can lead to peritonitis and the most common
hematuria, bright red urine with blood clots. Normal PE and VS causative agent of which is Staph aureus. FALSE (Strep. pneumo)
 Hematuria may be related to prior viral infection 6. Prognosis is said to be guarded for MPGN. TRUE
 Infection of the urinary bladder 7. IgA nephropathy is the most common chronic glomerular disease
worldwide. TRUE
INTERSTITIAL NEPHRITIS 8. IgA nephropathy will have an elevated serum elevated levels of C3.
 Glomerular injury associated with hypersensitivity reactions FALSE (normal)
especially to drugs 9. An uncommon manifestation of Henoch Schonlein Purpura is
 Caused by: Allergic reactions ureteritis. TRUE
10. Wilm's tumor is the second most common malignant abdominal
PERITONITIS AND SEPTICIMIA tumor in chlidhood. TRUE
 Most common agent: S. pneumoniae
MATCHING TYPE (please double check. Di ko alam kung tama mga sagot sa
VESICO URETHERAL REFLUX samplex eh)
 diagnosed by void cystourethrogram  Post-infectious GN – hematuria with clinical signs
 Benign familial hematuria – Asymptomatic hematuria without
LUMBOSACRAL MENINGOCOELE abnormalities
 renal imaging studies aside for an ultrasound is performed  Urolithiasis – hematuria with clinical signs
 Hemorrhagic cystitis – gross hematuria
VOIDING CYSTOGRAM  Exercise hematuria – Asymptomatic hematuria without
 Reflux up to ureter only: Grade 1 abnormalities
 Urinary tract infection – gross hematuria

5 ejg
 Acute kidney injury-failure – asymptomatic hematuria with  VCUG – vesicoureteral reflux
proteinuria Alternative drugs for nephrotic syndrome
 Cyclophosphamide – alopecia
MATCHING TYPE  Chlorambucil – hematologic malignancy
Clinical condition  Cyclosporine – gingival hypertrophy
 Nephrotic syndrome – hypervolemic hyponatremia Acute GN urinary findings
 Hypoaldosteronism – hypovolemic hyponatremia  Gross hematuria – 2-3 weeks
 Use of loop diuretics – hypovolemic hyponatremia  Microscopic hematuria – 12-18 months
 Congenital heart failure – hypervolemic hyponatremia  Proteinuria – 3-6 months
 Diabetes insipidus – NOTA
Intervention in hyperkalemia
 Calcium gluconate administration – protects the myocardium from
arrhythmia formation
 Inuslin glucose combination – shift potassium from the extracellular
fluid compartment to the intracellular compartment
 Hydration of the patient – dilutes serum potassium
 Kayexalate enema – gains sodium and excretes potassium at the GIT
 Diuretics – removes potassium from the body thus decreases total
body potassium stores
 Dialysis – removes potassium from the body thus decreases total
body potassium stores
 B2 agonist administration – shift potassium from the extracellular
fluid compartment to the intracellular compartment

MATCHING TYPE
Glomerular disease
 IgA nephropathy – recurrent gross hematuria without extrarenal
manifestation
 HSP nephritis – recurrent gross hematuria with extrarenal
manifestation
 Alport syndrome – persistent hematuria with sensorineural hearing
loss REFERENCE: Sandamakmak na
Imaging studies samplex. Plus Winstons compilation
 DMSA – renal scarring and Ayesha notes Goodluck and God
 DTPA – obstructive uropathy bless!

6 ejg
NEPHROLOGY A (MATTHEUS)

HEMATURIA
Glomerular​ if urinalysis (+) significant hematuria & proteinuria

GROSS MICROSCOPIC

Bright red blood, clots in urine, or tea-colored urine • >5 RBCs/hpf on more than two occassions
● Presence of ​clots​ usually suggest ​surgical, urological problem ​(e.g. ​postglomerular
disease​) • Significant hematuria: ​RBC in the urine is > 5 ​(Other literature, >3)
● Tea-colored urine usually suggests the ff: • Persistent hematuria : defined as ​three positive urinalyses
○ glomerular damage ➔ repeat urinalysis​ is usually done if results are ​equivocal
○ intake of drugs (e.g ​Metronidazole​) ➔ three positive urinalyses, based on dipstick and microscopic examination over a 2-3
○ hemoglobinuria week period of time because RBC passage is not constant
○ myoglobinuria ◆ Eg. 6 mos: + hematuria, no FH → monitor patient & no other sx of renal
progression before doing anymore intensive work up
◆ Eg. 10-20 rbc/hpf on 3 diff occ → repeat urinalysis & observe, important to ask
for FH of chronic kidney dse
➔ Test strips can detect 5-10 intact RBC’s/mul or 2-5 RBCs/hpf
◆ 2-5 RBCs/hpf is a positive test but does not necessarily mean significant
hematuria
◆ POINT: test strips (qualitative) should be correlated with microscopic findings
(quantitative)
➔ Benign Familial Hematuria is presented as having normal renal function tests with RBC
in the urine as the only abnormality.

➢ long term follow up is necessary to tule out progressive renal dse


➢ Correlates w/ best renal prognosis

24H urine creatinine clearance: most sensitive tool to assess renal fxn in stable state
24H urine albumin determination: one way to distinguish tubular vs glomerular proteinuria
MPGN → guarded → prognosis

Proctor’s Notes 1/10


NEPHROLOGY A (MATTHEUS)

Post-Infectious Glomerulonephritis Nephrotic Syndrome IgA Nephropathy

Definition Characterized by a ​previous infection ​prior to the onset of ISKD Guarded prognosis for other primary types
the nephritic syndrome
Most common chronic glomerular disease
Infection​ (pharyngitis, skin infection) → ​latent worldwide
(asymptomatic) → ​nephritic​ (all should be present)
Benign hematuria ​without systemic
manifestations

Characterized by a ​predominance of IgA within


mesangial
deposits of the glomerulus
➔ (+) microscopic or gross hematuria
➔ (-) systemic manifestations
Initially presents with significant microscopic
hematuria without HTN or edema. HTN and edema
occurs only as dse progresses.

Latent period (asymptomatic): Minimal Change Nephrotic Syndrome Natural history:


Establish presence to strengthen the dx of GN ➢ initial and persistent microscopic
Usually seen in YOUNG MALES (2-6 Y/O) hematuria​, then
Post -strep: 10-14 days ➢ gross hematura​, then
Viral: <week - 6weeks MOST STEROID RESPONSIVE ➢ eventually disappears, ​with URTI
Bacterial: 6wks - 3 mos Sensitivity of 93.1% (positivity)
Specificity of 72.2% (negativity) ➔ an ​immune complex disease ​caused by
Representative dse: abnormalities in IgA
Post-streptococcal glomerulonephritis Membranous GN ➔ familial clustering
Grp A Beta Hemolytic Streptococcus (GABS) → ​Can also Adult nephrotic syndrome ➔ genome-wide linkage analysis: 6q22-23
cause Rheumatic fever, treated separately ➔ Recurrence of ​gross​ hematuria whenevr px
suffers URTI
Case eg (subtract last 2 numbers sa case)
15 days PTA: sorethroat
4 days PTA: edema
Day 11: latent period

Pathogenesis 1. Immune mediated disease (circulating) MASSIVE PROTEIN LOSS IN URINE


a. Injury to kidney d/t deposition of circulating ➢ Main event that leads to cascade of
immune complex in the kidney → immune manifestation
complex dse! ➢ Clinical manifestations of nephrotic syndrome
2. Molecular mimicry primarily caused by this
3. In situ immune complex formation between anti-strep
abs & glomerular planted antigen (non-circ)
4. Direct complement activation by strep Ag in glomeruli
→ cause for depleted C3

Proctor’s Notes 2/10


NEPHROLOGY A (MATTHEUS)

Clinical CARDINAL MANIFESTATION: HEMATURIA ​(gross or CLINICAL HALLMARK: EDEMA FORMATION ​periorbital ➔ Westerners present with ​gross hematuria
manifestation microscopic) edema/facial edema: composed of loose tissue ➔ Asians (Japan) present with ​microscopic
Proteinuria Scrotal edema hematuria and/or proteinuria
Edema → periorbital, subsides then lower Pitting edema - pretibial edema ➔ Gross hematuria may occur in association
HTN with URTI or GI infection, loin pain​.
Oliguria If px have this, MOST APPLICABLE INITIAL DX STUDY → ➔ IgA nephropathy: ​Gross hematuria may recur.
Dyspnea, HF urinalysis ➔ Proteinuria ​is frequently in the nephritic range
Nephrotic proteinuria (<1000mg/24hr) ​in patients with asymptomatic,
Azotemia microscopic hematuria
Early mortality ◆ Indicator of prognosis → devt of
proteinuria
➔ Post-strep GN: Gross hematuria ​does not r​ ecur. ➔ Normal serum C3

Laboratory Urinalysis (24H urine determination): to diagnose Proteinuria (Albuminuria)


➢ gross/microscopic • Dipstick: ​4+ ​albumin
➢ Proteinuria: <40mg/m2/hr • Protein/Creatinine ratio: (random urine sample-screening
Serum Electrolytes (Na, K, Ca, Phosphorus) only)
ABG ➔ >200mg/mmol
Renal Function Test: BUN/Crea, eGFR ➔ >2mg/dL
➢ Estimated GFD is best used to assess renal ➔ Determine nephrotic range proteinuria
function by 24h urine sample ➔ Next best thing to do since timed urine collection
➢ Serum crea (mg/dl) / length x constant is difficult to do
ASO, neuramidase, DNAse
➢ ASO - most common method to document 24-hour urine albumin determination (Gold Standard)
previous infection ➔ Through body surface area: >40mg/m2/hr
➢ 80%: pharyngeal (minimum)
➢ 60%: S. pyoderma (lower accuracy if skin) ➔ Through weight:
➢ Sensitivity is lower >50mg/kg/day
➢ Strep infxn: inc after 2 weeks
Throat swab/culture Investigation of Initial Presentation
Skin lesion discharge culture CBC with platelet count
Serum C3, C4 Serum creatinine, cholesterol, TPAG (accdg to ISKDC)
Urinalysis
24-hour creatinine clearance / eGFR
Renal Ultrasound (not mandatory; only if entertaining DDx)
Others:
➔ C3 (depends on the clinical features) - prognostic
value
➔ Minimal Change Dse = normal C3
➔ Primary Type: Membranous Proliferative Dse =
Decreased C3
➔ Hepatitis B screening
➔ Tuberculin test

CXR Cardiomegaly
Equalization of vascular markings (basal & apical)
❖ BOTH suggest ​CONGESTION

Proctor’s Notes 3/10


NEPHROLOGY A (MATTHEUS)

Complications Volume overload Thrombosis


Renal failure
Electrolyte imbalace Hypovolemia - Emergency! Emergency!
Hypertensive crisis/encephalopathy Abdominal pain, oliguria, cold peripheries, poor pulse
➢ Complication that could have caused seizure: volume, hypotension, hemoconcentration, usually minimal
HTN edema
20-25% salt
1. Albumin 0.5-1.0g/kg/dose over 1-2 hours
infusion
2. Diuretics: Furosemide 1-2 mg/kg/dose

Hyperlipidemia

Infection​ - relapses with complaint of ​severe​ ​abd. pain;


primary peritonitis
1. cover for both gram (+) and (-) organisms until
cultures are available
2. prophylactic oral penicillin ​125-250 mg BID in
an edematous child
3. Step pneumoniae is most common agent
causing peritonitis and septicemia
4. Primary peritonitis is not full-blwon, (+) diarrhea
and discomfort

**peritoneal infection: severe abd pain, periumbilical w/


some abd guarding assoc with fever

Treatment Antibiotics: International Study of Kidney Disease in Treatment: NO STANDAR


mandatory regardless of the etiology Children(ISKDC) proper BP control
most viral causes have shorter latent period ➔ 8 weeks​ (4 weeks daily and 4 weeks alternate fish oil
prognosis of acute post strep GN is very good day therapy) ➔ with anti-inflammatory omega-3 FA
➔ 2 months intensive tx ➔ decrease rate of renal progression
PENICILLIN FOR 10 DAYS ➔ 1st half (4 weeks daily)​: ​60mg/m2 immunosuppressive therapy with corticosteroids or
Mandatory in px with sig hematuria & proteinuria, dec C3 & ➔ 2nd half (4 weeks alternate): 40mg/m2 more intensive
+ASO ➔ 4 wks: minimum duration of Tx with HIGH ➔ multidrug regimens (i.e.
dose steroid prior tapering cyclophosphamide) • beneficial in some
patients
Arbeitsgemeinschaft fur Padiatrische Nephrologie ACEis and ARBS
(APN) ➔ still under study concerning reduction of
➔ 12 weeks ​(6 weeks daily, 6 weeks alternate proteinuria and retarding renal
daily) progression
➔ 3 months intensive
➔ superior than ISKDC regimen Prophylactic antibiotics and tonsillectomy have
➔ 1st half (6 weeks daily): 60 mg/m2 no proven benefit.
➔ 2nd half (6 weeks alternate): 40mg/m2
➔ Associated with longer remission period & Renal transplantation (no absolute cure) or
fewer relapses dialysis

Allograft loss caused by IgA nephropathy in


DOC: ​Prednisone 15-30% of patients causes recurrence.
• For preteens: dose can be as high as 80mg/m2 (lecture) IgA Nephropathy ​with SYSTEMIC
Increased doses of steroids & prolonged duration are MANIFESTATIONS

Proctor’s Notes 4/10


NEPHROLOGY A (MATTHEUS)

important in reducing the risk of relapse

Acute Steroid-Sensitive NS Diet


1-2 wks ➢ Protein @ 1.5- 2.5 gms/kg
s/sx renal dysfxn provide adequate amount of protein intake for
Edema growth & nitrogen balance
HTN ➢ Salt restriction: 1-2 gms/day - no added salt
Pulmonary congestion ➢ Ensure physical activity and prevent excessive
If abnormal creatinine → + hematuria weight gain.
➢ Education regarding effects of ​high-dose
Diuretic steroids:​ ​voracious appetite, central obesity, fluid
2-3 wks retention, hypertension, DM, cataract
Oliguric to diuretic
+hematuria Edema
➢ Oral furosemide
Resolution/Recovery ➢ Spironalactone
3mos - 1 1/2yrs ➢ Albumin transfusion

Prognosis Good Good Check table above

Samplex/addnl APGN: Immunization **​cough, cold, low grade fever, too many rbc
notes Assoc with HTN, trace back 10 days for hx of infection, Live vaccines (measles, polio boosters) not given to
formation of immune complex patients on corticosteroids
➢ Can be given 4 weeks after cessation of
PSGN: Essentials for dx treatment
Evidence of previous or ongoing streptococcal infection Killed vaccine may be given anytime
(ASO) 23-polyvalent ​pneumococcal​ polysaccharide vaccine
➢ Conjugate pneumococcal: nephrotic px on steroid
**sudden onset facial edema, distention of abdomen, edema tx
of LE, tea-colored urine, oliguria, HA PVC 12
Influenza A
**facial edema, fluid wave, dec breath sounds, gr 2 pitting
Susceptible Nephrotics
**considered when urinalysis has sig. Hematuria & proteinuria Varicella exposure
➢ Zoster immunoglobulin within 72 hours from
** (+) micro hema: 10-20 rbc/hpf & proteinuria 2 → if thinking exposure
for glomerulonephr(I)tis → ask for previous (I)nfection ➢ Acyclovir, if varicella develops
➢ In steroid tx → exposed to varicella →
** false: complicated course is anticipated for post-viral GN varicella immunoglobulin immed!
• Measles exposure
• Gamma globulin

Proctor’s Notes 5/10


NEPHROLOGY A (MATTHEUS)

Henoch-Schonlein Purpura (HSP) Nephritis Renal Venous Thrombosis

➢ also referred to as Anaphylactoid Purpura


➢ small vessel vasculitis characterized by a ​TETRAD
1. purpuric rash (hallmark)
a. palpable purpura, symmetical, in gravity-dependent areas ​(lower extremities)
or pressure points (buttocks)
b. greater than macule; >1cm; elevatedfrom red, becomes purplish, to
yellow-brown, then disappears
2. Arthritis
a. big joints​ (wrist, elbow, knees, ankle)
b. signs of inflammation (​rubor, calor, dolor, tumor, functio laesa)​ • DDx:
SLE/RA/Septic arthritis/JRA (small joints, primarily)
3. abdominal pain
a. primary peritonitis
b. can present as an ​acute abdomen​; mistaken for appendicitis
4. Glomerulonephritis

HSP Nephritis and IgA Nephropathy have identical findings, except that systemic findings
are only found in HSP.
➔ IgA deposits in mesangial cells of glomerulus WITH SYSTEMIC manifestation

Pathogenesis - remains unknown Thrombus Formation


➔ appear to be mediated by the ​formation of immune complexes​ containing ​polymeric • mediated by ​endothelial cell injury ​resulting from hypoxia,
IgA1​ within ​capillaries of the skin, intestines, and glomerulus endotoxin, or contrast media
• endothelial cell injury causes reparative mechanisms
• vasoconstriction (more severe in​ clean cut)
• platelet plug
• roughening of capillary wall
• mechanical damage to RBC and subsequent lysis
• release of hemoglobin (toxic to the kidney)
• hemoglobinuria (tea-colored urine)
• renal failure

Diagnosis Diagnosis
Non-specific: ​based on clinical findings • suggested by the development of ​hematuria and flank masses​ in patients positive for
➔ gross hematuria: 20-30% predisposing clinical factors
➔ isolated microscopic hematuria • most patients also have ​microangiopathic hemolytic anemia (RBC lysis) and
➔ hematuria and proteinuria thrombocytopenia
➔ acute nephritic s/sx • UTZ: marked enlargement
➔ nephrotic s/sx • Radionucleotide studies: little or no renal function in affected
➔ renal insufficiency kidneys
Specific •​ Doppler Flow of IVC and renal vein confirm the diagnosis
➔ renal manifestations occurring ​up to 12 weeks after initial presentation •​ Contrast studies are avoided to minimize the risk of further vascular damage.

Ureteritis Manifestations
• ​uncommon urologic manifestation • heralded by ​sudden onset gross hematuria and unilateral/bilateral flank masses
• loin pain and renal colic • may also present with ​microscopic hematuria, flank pain, hypertension and oliguria
• children <5 years • ​RVT​ is usually​ unilateral.
• may lead to ​ureteral stenosis (bilateral) ​ and structures, causing • hemoglobinuria damages kidney, but no failure

Proctor’s Notes 6/10


NEPHROLOGY A (MATTHEUS)

hydronephrosis ​that requires ​surgical correction •​ Bilateral RVT ​causes ​acute renal injury​ and immediate ARF.
• Hydonephrosis will destroy kidneys at whatever degree. Differential Diagnosis
• Hemolytic-Uremic Syndrome
• secondary to undercooked hamburgers; older children • (+) sudden onset gross hematuria
• (+) microangiopathic hemolytic anemia
• (+) thrombocytopenia
• (+) Flank Masses
• Hydronephrosis
• Polycystic Kidney Disease • Wilm’s tumor
• Renal abscess
• Hematoma

Treatment Treatment of PRIMARY CAUSE


• ​No controlled data • supportive care:
• Uncontrolled studies: high potential valye of high-dose ​steroids​, • correction of fluid and electrolyte imbalance • treatment of renal insufficiency
tacrolimus, ​cytotoxic​ therapy (i.e. cyclophosphamide, • Anticoagulation or thrombolysis remain controversial
azathioprine) in patients with crescentic GN or sig. proteinura. ➔ steptokinase
• Additional ​dipyridamole ​and/or ​heparin/warfarn​ may provide ➔ Urokinase: lyse thrombus: regain BF
added benefit. ➔ recombinant t-PA
➔ difficult in patients with sepsis and deranged coagulation
• Surgical Thrombectomy - thrombosis of the IVC • Nephrectomy
• children with severe HTN (kidneys non-functional) refractory to antihypertensive medications

Prognosis Prognosis
• generally favorable ● Perinatal mortality from RVT has decreased significantly due to intensive support.
• s/sx may continue for several months, especially GN ● Partial or complete renal atrophy is a common sequela of RVT leading to renal
• risk of chronic renal insufficiency is 2-5% insufficiency (early onset), renal tubular dysfunction and systemic hypertension
• BEST PROGNOSIS - isolated microscopic hematuria ● Recovery of renal function is common in children with RVT due to nephrotic syndrome or
• Highest risk of CHRONIC RENAL FAILURE: ​both acute nephritic and nephrotic syndromes cyanotic heart disease
● RVT during neonatal period requires follow-up for life because of early onset
hypertension.

Proctor’s Notes 7/10


NEPHROLOGY A (MATTHEUS)

URINARY TRACT INFECTION

➔ E. coli: most common etiologic agent


➔ If with posterior midline anomaly → diagnostic
imaging (advanced renal imaging)
➔ Clean catch is not reliable if >100,000 + 2 organisms
➔ KUB: utz, kidney, urinary bladder
➔ Lower tract infection: AMOXICILLIN

Radiologic Imaging of the GUT


Radiotype imaging DMSA Scan detects:
➔ scars after pyelonephritis
➔ renal agenesis
➔ ectopic kidney
Static renal exam
Specific to the CORTEX

Eg. abd pain, pain on urination, lower abdominal tenderness, R


sided costovertebral angle tenderness
➔ Urine culture & sensitivity: confirm dx

Eg. High grade fever, dark colored urine, + pus cell, 50-60hpf, +
nitrite & WBC esterase
➔ Next best step: ​URINE CULTURE (GOLD
STANDARD)
Eg. + nitrite & WBC esterase, microscopy
➔ Highest possibility that UTI is present

Eg. better sensitivity for diagnosisng TRUE UTI


➔ Pyuria
➔ Bacteriuria
➔ WBC esterase test
➔ Nitrite test

Proctor’s Notes 8/10


NEPHROLOGY A (MATTHEUS)

WILMS TUMOR (NEPHROBLASTOMA)

➔ Complex mixed embryonal neoplasm of the kidney


composed of;
◆ Blastema
◆ Epithelia
◆ Stroma
➔ 8 cases/million children <15 years of age
➔ Peak: 2-5 years of age
➔ Second most common malignant abdominal tumor in
childhood
➔ Abdominal masses are considered a medical
emergency.
➔ May​ arise in one or both kidneys​; one side may be
bigger than the other. The incidence of a bilateral Wilms
is 7%.
➔ May be associated with the following:
◆ Hemihypertrophy ​- one side is bigger, very
obvious o ​Aniridia ​- absence of iris, unilateral or
bilateral
◆ Other​ congenital anomalies​, usually of GUT
◆ variety of syndromes
➔ Majority are sporadic
➔ 1-2% are familial, AD
◆ Familial cases are associated with diagnosis at an
earlier age
◆ increased frequency of bilateral disease
◆ congenital absences are usually absent
◆ not associated with Wilms tumor gene (WT1)

Clinical Manifestations Diagnosis: Treatment


• abdominal mass - approached as malignant mass •​ Abdominal mass - urgent nature of the condition → • Surgery
• smooth, firm approached as malignant • Patency of the IVC should be established prior to resection
•​ occassionally cross the midline • complete PE, CBC • ​IVC not patent: chemotherapy should be administered
• abdominal pain, vomiting • Liver and Kidney FT’s
• hematuria • Specific tumor markers
• hypertension • Flat plate of the abdomen
•​ Ultrasonography - often used
• CT/MRI
• Chest X-ray: detection of mets., useful for pre-op

Proctor’s Notes 9/10


NEPHROLOGY A (MATTHEUS)

Prognostic Factors BENIGN FAMILIAL HEMATURIA


➔ Verify if hematuria present also in immediate or distant
Major prognostic factors are ​tumor size, stage and histology family members
Prognosis is worse with: ➔ Work up minimally then observe (Urinalysis, Rnela
➔ large tumor ​(>500g) function test and UTZ) then follow up
➔ advanced stage (S3, S4) ➔ If there is no progression, no need for renal biopsy
➔ unfavorable histological type
ALPORT’S SYNDROME
Wilms tumor constitutes a paradigm of successful ➔ • familial
multidisciplinary treatment; more than ​60% of patients with all ➔ initially a seemingly benign micro hematuria →
stages generally survive without systematic manifestation in children but
may persist in adulthood with assoc.
Stages I through III have a cure rate varying from 88-98%. Sensorineural deafness
➔ after several years: ​proteinuria, renal deterioration
➔ In family hx: one relative would have progressive
➔ renal problems ending up in dialysis
➔ associated sensorineural deafness

Proctor’s Notes 10/10

You might also like